addthis

feed twitter facebook

1.2.08

Pathophysiology Tricky Questions asked in saudi medical council exam

Which of the following cells play a crucial role in the pathogenesis of alveolar-capillary damage in adult
respiratory distress syndrome (ARDS)?


A. CD4-positive lymphocytes

B. CD8-positive lymphocytes

C. Eosinophils

D. Mast cells

E. Neutrophils


Explanation:

The correct answer is E. ARDS, pathologically referred to as diffuse alveolar damage, is a clinical syndrome of
acute respiratory failure resulting from diffuse injury to the alveolar/capillary barrier. Such injury may be caused
by a great variety of initiating insults, the most frequent of which are shock, severe trauma, sepsis, and gastric
aspiration. All these different forms of injury result in recruitment of neutrophils within the alveolar capillaries.
Neutrophils release chemokines that attract histiocytes and produce oxygen radicals, prostaglandins, and
proteases that damage alveolar epithelium. Formation of hyaline membranes is due to a combination of plasma
fluid extravasation and alveolar cell necrosis.

CD4+ (helper) lymphocytes (choice A), CD8+ (cytotoxic) lymphocytes (choice B), eosinophils (choice C), and
mast cells (choice D) have been implicated in a number of pulmonary diseases, but not in diffuse alveolar
damage.



An animal is made diabetic by injection of a drug that destroys pancreatic β cells. Removal of which of the
following organs would most likely produce a decrease in blood glucose concentration in this animal?


A. Anterior pituitary

B. Colon

C. Gonads

D. Kidney

E. Pancreas


Explanation:

The correct answer is A. Two of the secretions of the anterior pituitary affect the sensitivity of peripheral tissues
to the action of insulin. Growth hormone has a direct effect on liver and muscle to decrease insulin sensitivity.
This may be partly through a growth hormone-induced decline in insulin receptors or to unknown post-receptor
defects. In excess, growth hormone is "diabetogenic," and about 25% of patients with acromegaly have
diabetes. ACTH indirectly has anti-insulin effects by virtue of the cortisol secretion it evokes. Like growth
hormone, cortisol also decreases insulin sensitivity in peripheral tissues. A third anterior pituitary hormone,
TSH, also tends to increase blood glucose levels. In this case, the effect is probably mediated mostly through
increased glucose absorption by the gut. Patients with hyperthyroidism can sometimes exhibit a postprandial
glucosuria because of excessive intestinal glucose absorption. In diabetic animals, the removal of the anterior
pituitary may lower blood glucose by increasing tissue sensitivity to whatever insulin remains.

Removal of the colon (choice B) should have little effect on blood glucose since dietary glucose is absorbed in
the small intestine.

Sex steroids secreted by the gonads (choice C) have little effect on blood glucose concentration.

The kidney (choice D) plays an important role in reabsorbing filtered glucose. In diabetes, the tubular
reabsorption maximum is exceeded and glucose spills over into the urine. The loss of glucose in the urine helps
to reduce the severity of the plasma hyperglycemia. Removal of the kidneys would, if anything, make the
hyperglycemia worse.

Pancreatectomy (choice E) would make the hyperglycemia worse by removing the source of any remaining
insulin.

A 33-year-old woman gives birth to a baby girl. The next day, she begins to bleed from her vagina and from
venipuncture sites. Laboratory studies demonstrate decreased platelets, prolonged prothrombin time (PT) and
partial thromboplastin time (PTT), and increased fibrin split products. These features are most consistent with
which of the following?


A. Disseminated intravascular coagulation

B. Hemophilia A

C. Severe liver disease

D. Vitamin K deficiency

E. Von Willebrand's disease


Explanation:

The correct answer is A. The patient is experiencing disseminated intravascular coagulation (DIC), a feared,
and often life-threatening complication of many other disorders, including amniotic fluid embolism, infections
(particularly gram-negative sepsis), malignancy, and major trauma. The diagnosis is suspected when both a
decrease in platelets and a prolongation of PT and PTT times are observed. The observed hematologic
abnormalities are due to consumption of platelets and clotting factors, caused by extensive microclot formation
with accompanying fibrinolysis (reflected by the increased fibrin split products). The D-dimer assay measures
cross-linked fibrin derivatives, and is a specific test for fibrin degradation products.

Hemophilia (choice B) will alter the PTT without affecting the other indices.

Severe liver disease (choice C) produces alterations comparable to those in vitamin K deficiency, and platelets
can also be decreased secondary to a generalized metabolic marrow dysfunction, but fibrin split products would
not be increased.

Vitamin K deficiency (choice D) is associated with alterations in both PT and PTT, but platelets will not be
decreased, nor will fibrin split products be increased.

Von Willebrand's disease (choice E) produces impaired platelet adhesion and increases the bleeding time as
well as the PTT, but will not produce the other features described.



A 54-year-old woman presents with a pansystolic murmur along the lower left sternal border radiating rightward to
the midclavicular line. The murmur is medium pitched, has a blowing quality, and increases slightly on inspiration.
An S3 is audible along the lower left sternal border. Jugular venous pressure is elevated, and a prominent "v"
wave is visible. Which of the following is the most likely etiology of the S3?


A. Aortic stenosis

B. Mitral regurgitation

C. Pulmonic stenosis

D. Tricuspid regurgitation

E. Volume overloaded left ventricle

F. Volume overloaded right ventricle


Explanation:

The correct answer is F. The origin and radiation of the pansystolic murmur suggest tricuspid valve
incompetence. This is further supported by its pitch and quality, and by the fact that it increases on inspiration
when cardiac volume increases. The regurgitant blood flow from the ventricle during systole increases jugular
venous blood pressure and atrial v wave amplitude. The origin of the S3 sound, which occurs during early rapid
filling, is the filling of a volume-overloaded right ventricle. The right ventricle overload is caused by the
combination of systemic venous return and the return of the regurgitated blood volume into the right ventricle.
Right ventricular failure and dilatation, with enlargement of the tricuspid valve orifice, is the most common cause
of tricuspid regurgitation and is often secondary to pulmonary hypertension or left ventricular failure.

Aortic stenosis (choice A) causes a harsh, shrill, midsystolic, crescendo-decrescendo murmur and would not
necessarily elevate right heart (and so jugular venous) pressures. It is often associated with an S4 (late rapid
filling) rather than an S3.

Mitral regurgitation (choice B) causes a soft, blowing, pansystolic murmur and is associated with an S3. It would
elevate left atrial pressures, not right atrial (and so jugular venous) pressures.

Pulmonic stenosis (choice C) would also cause a crescendo-decrescendo murmur, not an S3.

Tricuspid regurgitation (choice D) is the source of the murmur but not the source of the S3 sound.

A volume-overloaded left ventricle (choice E) could cause an S3, but tricuspid regurgitation will not cause a
volume overload in the left ventricle.




A routine physical examination demonstrates hypercalcemia in a 40-year-old man. Circulatory levels of
parathyroid hormone are also elevated. Exploratory surgery of the neck reveals diffuse hyperplasia of all four
parathyroid glands. Which of the following screening studies would most likely be helpful in confirming the
diagnosis of multiple endocrine neoplasia, type I (MEN I)?


A. Pentagastrin-simulated calcitonin secretion

B. Serum epinephrine

C. Serum gastrin

D. Serum norepinephrine

E. Urinary adrenaline:noradrenaline ratio


Explanation:

The correct answer is C. In multiple endocrine neoplasia, type I (MEN I), parathyroid hyperplasia and/or
adenomata are associated with pancreatic and duodenal endocrine tumors and endocrine hyperplasia. The
most common secretory product of the pancreatic and duodenal endocrine lesions is gastrin. Pituitary
adenomata may also occur in MEN I.

Pentagastrin-stimulated calcitonin secretion (choice A) is a marker for medullary carcinoma of the thyroid, which
is a component of MEN II.

Epinephrine (adrenaline) is produced by pheochromocytomas that occur as part of MEN II; serum epinephrine
levels (choice B) and urinary adrenaline:noradrenaline ratios (choice E) may be used for screening for
pheochromocytomas.

Serum norepinephrine (choice D) is usually not helpful in the diagnosis of pheochromocytoma, since serum
catecholamines can be increased by hypoglycemia, strenuous exertion, and central nervous system disease.


A 40-year old man complains of increasing difficulty in swallowing over the past 3 years. He reports a feeling of
pressure in his chest occurring 2-3 seconds after swallowing a solid bolus. He also experiences regurgitation of
undigested food eaten hours previously. A radiograph taken after swallowing barium shows a distended
esophageal body with a smooth tapering at the lower esophageal sphincter. Manometry shows the absence of
esophageal peristalsis with swallowing and a lower esophageal sphincter that fails to relax. What is the most likely
diagnosis?


A. Achalasia

B. Diffuse esophageal spasm

C. Incompetent lower esophageal sphincter

D. Oropharyngeal dysphagia

E. Scleroderma


Explanation:

The correct answer is A. Achalasia is an acquired esophageal motility disorder that slowly develops. The motility
is abnormal due to the loss of inhibitory enteric neurons of the esophageal body and lower esophageal
sphincter. Both vasoactive intestinal peptide and nitric oxide function as inhibitory neurotransmitters here, and
the presence of both is decreased in achalasia. Radiographs typically show a dilated esophagus that tapers at
the lower esophageal sphincter, producing a so-called "bird's beak." Because of the poor motility, ingested food
is regurgitated and can lead to aspiration symptoms. Heartburn can occur due to production of lactic acid in the
esophagus as the retained ingestate is fermented. Manometric demonstration of absent peristalsis in the
esophageal body and poor relaxation of the lower esophageal sphincter with a swallow confirm the diagnosis.

The primary complaint with diffuse esophageal spasm (choice B) is mid-sternal pain that can be misdiagnosed
as cardiac pain. The pain is caused by prolonged contraction of the entire esophageal body. Symptoms can be
brought on by eating certain hot or cold meals. A manometric study may show poor peristalsis in the smooth
muscle portion of the esophageal body, but lower esophageal sphincter function is unaffected.

The primary complaint with incompetent lower esophageal sphincter (choice C) is heart burn and regurgitation
due to gastroesophageal reflux. Endoscopic examination of the esophagus may reveal inflammation, erosions,
and even ulcers. A manometric study would show lower-than-normal resting tone in the lower esophageal
sphincter, or a sphincter that relaxes inappropriately.

The fact that the patient's symptoms do not occur until 2-3 seconds after a swallow suggests that
oropharyngeal dysphagia (choice D) is not the diagnosis. The presence of cough, hoarseness, or nasal
regurgitation commonly occurs with this disorder. Oropharyngeal dysphagia is often due to neurological or
muscle disorders like stroke, amyotrophic lateral sclerosis, muscular dystrophy, or myasthenia gravis.

Scleroderma (choice E) is a connective tissue disease in which esophageal smooth muscle is gradually
replaced by dense collagenous material. Manometry would show poor esophageal peristalsis and decreased
lower esophageal sphincter tone. Significant acid reflux with resultant esophagitis is almost universal.




Q2 DIAGARM[IMG]file:///C:/DOCUME%7E1/Owner/LOCALS%7E1/Temp/msohtml1/01/clip_image001.jpg[/IMG]



A 42-year-old-woman is admitted to the hospital because of syncopal attacks and difficulty breathing. She had
undergone mitral valve replacement 8 years ago. X-ray shows pulmonary congestion and an enlarged heart. A
phonocardiogram did not show a systolic murmur. Pressure tracings from the aorta, left ventricle, and left atrium
are shown in the diagram. Which of the following diagnoses best accounts for these findings?


A. Aortic regurgitation

B. Aortic stenosis

C. Mitral regurgitation

D. Mitral stenosis


Explanation:

The correct answer is D. It is evident that the patient has mitral obstruction because the left atrial pressure is
greater than the left ventricular pressure toward the end of diastole, when blood is flowing from the left atrium into
the left ventricle. The mitral valve replaced 8 years ago had undergone thrombosis resulting in obstruction of the
mitral orifice. The very high left atrial pressure resulting from the thrombosed valve has caused pulmonary
edema, which accounts for the dyspnea.

The aortic pressure and left ventricular pressure tracings are nearly superimposed during systole in the diagram,
which eliminates the possibility of aortic regurgitation (choice A) or aortic obstruction (choice B).

Mitral regurgitation (choice D) is characterized by a greatly elevated left atrial pressure toward the end of systole.
The increase in pressure is caused by backward flow of blood from the left ventricle into the left atrium through
the leaky mitral valve. The leak occurs during systole, and is characterized by a systolic murmur, which was not
noted in the patient. The left atrial pressure is normal at the end of diastole with mitral regurgitation because
blood flows unimpeded from the atrium into the ventricle when the mitral valve is open.



A patient with complaints of somnambulism has fallen asleep. She passes from light sleep into a deeper sleep.
Just before she experiences an episode of somnambulism, her electroencephalogram is likely to show


A. alpha waves

B. beta waves

C. delta waves

D. sleep spindles and K-complexes

E. theta waves


Explanation:

The correct answer is C. Delta waves are low-frequency, high-amplitude waveforms that herald the arrival of the
deepest type of non-REM sleep, stage 4 sleep. It is during this stage of sleep that somnambulism (sleepwalking)
occurs.

Alpha waves (choice A) are characteristic of relaxed wakefulness.

Beta-like activity (choice B) is characteristic of either alert wakefulness or REM sleep.

Sleep spindles and K-complexes (choice D) are characteristic of stage 2 sleep, which is a deeper sleep that
occupies roughly 45% of the sleep cycle.

Theta waves (choice E ) are characteristic of light sleep (stage 1).



A 51-year-old male smoker presents with fever and a cough productive of greenish-yellow sputum. The patient
states that he has had a morning cough with excessive mucus production for the past 5 years. Which of the
following abnormalities would most likely be found in this patient?


A. Apical cavitary lesions on x-ray

B. Curschmann spirals in his sputum

C. Elevated salt levels in his sweat

D. Enlarged hilar lymph nodes on x-ray

E. Increased Reid index


Explanation:

The correct answer is E. This patient presents with symptoms suggestive of acute infection (elevated
temperature, greenish-yellow sputum) on a background of chronic bronchitis, which is common in smokers.
Hyperplasia and hypertrophy of mucous glands in chronic bronchitis causes them to be present at deeper
levels in the bronchial wall than usual. The ratio of the gland depth to the total thickness of the bronchial wall is
termed the Reid index, which would be increased in this patient.

Apical cavitary lesions (choice A) might be indicative of cavitary tuberculosis. This condition is not associated
with excessive mucus production. Hemoptysis and weight loss might also be expected as clinical findings.

Curschmann spirals (choice B) are found in asthmatic patients and represent mucus casts of small airways.
This patient does not have the typical episodic history of acute asthmatic attacks with acute dyspnea as the
major clinical problem.

Elevated sodium chloride levels in sweat (choice C) are present in cystic fibrosis. This condition has an onset in
early life and is associated with excessive production of thick mucus, which predisposes to infection of the
airways and permanent damage.

Enlarged hilar lymph nodes (choice D) might suggest bronchogenic carcinoma or a granulomatous process,
which would be less likely than chronic bronchitis. In addition, patients with carcinoma often present with
hemoptysis and weight loss, rather than excessive mucus production.



A neurological examination of a 47-year-old woman reveals a normal corneal reflex in her right eye, but no
consensual corneal reflex in her left eye. Which of the following additional findings might be expected?


A. Absence of pupillary light reflex of the left eye

B. Hyperacusis of the left ear

C. Inability to abduct the right eye

D. Loss of pain and temperature of the left face

E. Loss of taste from the anterior two-thirds of the right tongue

F. Ptosis of the left eye


Explanation:

The correct answer is B. The first trick to this question is to determine where the lesion is. The corneal reflex is
tested by touching a cotton wisp to the eye. A normal response would be blinking of the ipsilateral eye as well
as the contralateral eye (consensual reflex). The afferent limb of the corneal reflex is contained within the
ophthalmic division of the ipsilateral ophthalmic nerve (V1), the efferent limb is by both (right and left) facial
nerves (VII). This woman had a normal corneal reflex in her right eye, indicating a normal right V1 and right VII.
However, she lacked a consensual reflex, indicating an abnormal left VIIth nerve. The next trick to this question
is to determine what other signs a lesion in the left VIIth nerve could produce. A lesion in the left VIIth would also
produce hyperacusis (increased sensitivity to sound) in the left ear because of paralysis of the stapedius
muscle, which ordinarily dampens sound transmission through the middle ear.

The absence of a pupillary light reflex of the left eye (choice A) could be caused either by a lesion of the left
optic nerve (CN II; afferent limb) or by a lesion of the left oculomotor nerve (CN III; efferent limb).

The inability to abduct the right eye (choice C) could be caused by a lesion of the right abducens nerve (CN
VI), which innervates the lateral rectus muscle.

Loss of pain and temperature of the left face (choice D) could be caused by a lesion of the spinal nucleus of V.
This nucleus is located in the medulla, and receives pain and temperature information from the face via the
trigeminal nerve (CN V).

Loss of taste from the anterior two-thirds of the right tongue (choice E) could result from a lesion of the right CN
VII.

Ptosis of the left eye (choice F) could result from a lesion of the left oculomotor nerve (CN III) because of
denervation of the levator palpebrae muscle. A lesion of the left VII would result in the inability to close the left
eye.



A 29-year-old woman with a history of irregular menses becomes amenorrheic. She had no problems conceiving
her first child at the age of 23, but she has been trying unsuccessfully to become pregnant for the past two
years. She also notes a weight gain of about 3 kg, increasing fatigue, puffy face and marked cold intolerance. A
complete blood count (CBC) reveals a Hb of 11.1 and an MCV of 90. Physical exam reveals a moderate-sized
diffuse enlargement of the thyroid gland. Which of the following thyroid profiles would most likely be seen in this
woman?


A. Low T3, low T4, high TSH

B. Low T3, low T4, low TSH

C. Low T3, high T4, low TSH

D. High T3, low T4, low TSH

E. High T3, high T4, low TSH


Explanation:

The correct answer is A. This woman is experiencing signs and symptoms of hypothyroidism, the most common
cause of which is chronic thyroiditis, or Hashimoto's thyroiditis. It is an autoimmune disorder, mostly affecting
women, in which antithyroid antibodies are produced. The immune response results in autoimmune destruction
of the thyroid gland, rendering it less able to produce thyroid hormone, causing hypothyroidism. Some other
symptoms of hypothyroidism are weakness, fatigue, coarse hair, constipation, hoarseness, and hearing loss.
Since the thyroid cannot produce thyroid hormone, both T3 and T4 would be low. In trying to compensate for
low thyroid hormone levels, the pituitary gland releases excess TSH to stimulate the thyroid gland to make more
thyroid hormone.



Q7[IMG]file:///C:/DOCUME%7E1/Owner/LOCALS%7E1/Temp/msohtml1/01/clip_image002.jpg[/IMG]


The work diagrams in the figure above show changes in left ventricular volume and pressure during one cardiac
cycle from a normal heart (diagram A) and following aortic valvular disease (diagram B). Which of the following is
expected to be increased in the heart depicted in diagram B as compared to the normal heart depicted in diagram
A?


A. Coronary artery oxygen content

B. Coronary blood flow during diastole

C. Coronary blood flow during systole

D. Coronary vein oxygen content

E. Myocardial oxygen tension (pO2)


Explanation:

The correct answer is B. Coronary blood flow is regulated almost entirely by the metabolic requirements of the
cardiac muscle. The heart depicted in diagram B has aortic stenosis. The peak systolic pressure of the left
ventricle has increased from a normal value of about 125 mm Hg to about 190 mm Hg. This increase in systolic
pressure has increased the stroke work output of the heart depicted by diagram B. The stroke work output is
equal to the area enclosed by the volume-pressure diagram. This increase in stroke work output increases the
oxygen consumption of the heart, thereby decreasing the content of oxygen in the venous effluent flowing from
the heart (choice D) as well as the oxygen tension (pO2) in the myocardium (choice E). It should be clear that
increasing myocardial oxygen consumption will not affect the amount of oxygen in the blood (arterial oxygen
content, choice A) entering the heart through the coronary arteries.

The heart normally uses about 70% of the oxygen in the arterial blood flowing through the coronary circulation.
Because there is not much oxygen left in the blood, increases in blood flow are required to supply the heart with
additional amounts of oxygen, i.e., oxygen extraction cannot be increased to a large extent. One other problem is
that blood flow falls to low levels during systole because the coronary blood vessels are compressed by the
contracting muscle. The increase in peak systole pressure caused by aortic stenosis compresses the coronary
vessels even more than normal, causing systolic blood flow to decrease greatly (choice C). This decrease in
systolic flow coupled with the increase in myocardial oxygen consumption that occurs with aortic stenosis causes
blood flow to increase greatly during diastole.



A child is 2 standard deviations below the expected mean height for his age. He also has delayed bone
maturation and a goiter. Analysis of genetic material reveals a point mutation in the thyroid hormone receptor.
Which of the following laboratory results would be expected in this patient?


A. Decreased radioactive iodine uptake test (RAIU)

B. Decreased resin T3 uptake test

C. Decreased plasma TSH concentration

D. Increased basal metabolic rate (BMR)

E. Increased plasma T4 concentration


Explanation:

The correct answer is E. Generalized resistance to thyroid hormone is a rare genetic abnormality (Refetoff's
syndrome). It results from mutations of the thyroid hormone receptor gene. Depending on the severity of the
disorder, patients may only be mildly affected or may exhibit striking hypothyroid-like symptoms including
decreased BMR (not increased, choice D). Growth can be stunted, there may be deaf mutism, and attention
span may be short. Because the thyroid hormone resistance is generalized, the normal negative feedback
effects of T4 and T3 at the hypothalamus and pituitary are also deficient. This would lead to an increased
plasma TSH concentration (not decreased, choice C). Because of the increase in plasma TSH, iodine trapping
by the thyroid follicular cells will be increased, leading to an increase in RAIU (not decreased, choice A) and an
increase in serum T4. Because of the increase in serum T4, the equilibrium between T4 and thyroxine binding
globulin (TBG) will be shifted toward increased bound T4 with a concomitant decrease in free TBG binding
sites. In the resin T3 uptake test, the added radioactive T3 would thus preferentially bind to the resin and not
the TBG, producing an increase in resin T3 uptake (not decrease, choice B).



A morbidly obese (450 lb) individual presents to the emergency department in respiratory distress. Arterial blood
gas studies show a PCO2 of 55 mm Hg, a PO2 of 60 mm Hg, and a pH of 7.28. Chest X-ray films are
unremarkable, with no evidence of emphysema, tumor, fibrosis, pulmonary infarction, or other disease.
Auscultation reveals a rapid but regular heart beat, and the pulse is strong. Which of the following is the most
likely explanation of the patient's arterial hypoxemia?


A. Decreased capacity for pulmonary diffusion

B. Decreased surface area of alveolar capillary membranes

C. Hypoventilation of central origin

D. Hypoventilation of peripheral origin

E. Inequalities of ventilation and perfusion


Explanation:

The correct answer is D. It is conceptually worth subclassifying hypoxemia in terms of the groups of the causes
listed in the answers. In this case, the patient has no evidence of primary pulmonary or cardiovascular disease,
and is known to be morbidly obese. Morbidly obese individuals are vulnerable to the Pickwickian syndrome
(after a character in a Dickens novel), in which pressure from a fatty neck causes intermittent airway
obstruction. The many other causes of hypoventilation of peripheral origin include suffocation, submersion,
skeletal abnormalities, trauma, phrenic nerve paralysis, polio, and tetanus.

Causes for decreased capacity for pulmonary diffusion (choice A) of 02 include processes such as respiratory
distress syndrome, emphysema, pulmonary fibrosis, and some granulomatous processes such as sarcoidosis.

Causes for decreased surface area of alveolar capillary membranes (choice B) include resection or
compression of the lung and emphysema.

The usual cause of hypoventilation of central origin (choice C) is respiratory center depression by morphine or
barbiturates.

Causes for inequalities of ventilation and perfusion (choice E) include chronic bronchitis, asthma, emphysema,
bronchiectasis, some granulomatous processes, and tumors.



Pulmonary artery pressure
45/25 mm Hg
Pulmonary wedge pressure
30 mm Hg
Left ventricular pressure
120/5 mm Hg
Aortic pressure
120/80 mm Hg


Cardiac catheterization was performed on a 51-year-old woman because of a 9-month history of worsening
fatigue and shortness of breath. What is the most likely diagnosis based on the pressures shown above?


A. Aortic regurgitation

B. Aortic stenosis

C. Mitral regurgitation

D. Mitral stenosis


Explanation:

The correct answer is D. The pulmonary wedge pressure (which is used as an estimate of left atrial pressure) is
elevated to 30 mm Hg and the pulmonary artery pressure is elevated to 45/25 mm Hg. The left ventricular
end-diastolic pressure is normal but is not equal to the pulmonary wedge pressure. A pressure gradient of 25
mm Hg (30 - 5) across the mitral valve is a clear indication of stenosis. The fatigue and shortness of breath
result from mild pulmonary edema caused by the increase in pulmonary capillary pressure. One can surmise
that the pulmonary capillary pressure is elevated because pressures are elevated at the arterial and venous
ends of the pulmonary circulation.

In aortic regurgitation (choice A), blood flows backward through the aortic valve during diastole when the valve
is closed. Left ventricular end-diastolic pressure (and pulmonary wedge pressure) may be elevated with chronic
aortic regurgitation once the myocardium has failed, but aortic regurgitation itself will not result in a pressure
gradient across the mitral valve.

In aortic stenosis (choice B), the blood is ejected from the left ventricle into the aorta through a
smaller-than-normal opening. Because the resistance to ejection of blood is high, the left ventricular pressure
increases greatly with normal systolic pressure in the aorta.

Mitral regurgitation (choice C) means backward flow of blood through the mitral valve during systole. This
accumulation of extra amounts of blood in the left atrium during ventricular systole leads to an elevation in the
pulmonary wedge pressure (which is used as an estimate of left atrial pressure).



A patient comes in to the doctor because of a chronic cough. He notes occasional streaks of blood in his sputum.
Chest x-ray reveals multinodular, cavitating lesions in the apical posterior segments of both lungs with evident
satellite lesions. The condition described is likely to occur in the apices of the lungs because they


A. are better perfused than the base

B. are more acidic than the base

C. contain more alveolar macrophages than the base

D. have a higher PO2 than the base

E. ventilate better than the base


Explanation:

The correct answer is D. The presentation is typical for reactivation pulmonary tuberculosis. The patient may
also note fever, malaise, and weight loss. The high PO2 found in the upper portion of the lungs provides a
favorable environment for growth of Mycobacterium tuberculosis, leading to reactivation tuberculosis. (In
contrast, primary tuberculosis tends to occur in the lower and middle lobes, where small infectious particles are
most likely to lodge after being inhaled.)

Ventilation increases from the top to the bottom of the lung, so choice E is wrong. Perfusion increases even
more rapidly than ventilation, so choice A is also wrong. As a result, the ventilation-perfusion ratio decreases
from the top to the bottom of the lung. The higher ratio at the apex of the lung results in a relatively elevated
PO2 at that location.

The apex of the lung has a higher pH than the base, so choice B is wrong. Because the ventilation-perfusion
ratio is higher at the apex, PCO2 would be lower, thus increasing the pH.

Regional differences in the density of alveolar macrophages (choice C) are not known to cause the described
predisposition.



A 35-year-old female, hospitalized after a motor vehicle accident, develops acute gastric stress ulcers. Increases
in which of the following normal physiological parameters may have contributed to this condition?


A. Bicarbonate transport

B. Epithelial regenerative capacity

C. Mucosal blood flow

D. Mucus secretion

E. Pepsin production


Explanation:

The correct answer is E. Pepsin production is a normal physiologic activity of the stomach that, in conditions of
stress, may overwhelm the stomach's weakened defenses and result in gastric ulceration. Gastric acid
production is another condition that may increase and cause acute ulceration. Furthermore, these two factors
may remain unchanged and still result in gastric ulcers if the gastric defenses are weakened by stress. All of the
other choices represent normal defensive forces in the stomach.

Increased bicarbonate transport (choice A) would protect the gastric epithelium from the potentially harmful
acidity of the gastric contents. The adherent mucus is relatively alkaline, providing local protection to the
superficial mucosa.

Gastric epithelial cells can normally replicate rapidly, allowing mucosal defects to be rapidly repaired. Increasing
the regenerative capacity of the epithelium (choice B) would have a protective effect against ulceration.

The gastric mucosa is richly supplied with blood, providing the epithelial cells with an ample supply of nutrients,
oxygen, and bicarbonate to contend with the harsh gastric microenvironment. Stress ulcers are associated with
compromised gastric blood flow, not increased flow (choice C).

Mucus protects the gastric epithelium by virtue of being water insoluble, impermeable to pepsin, and slowly
permeated by acid (H+). Increasing mucus production (choice D) has a protective effect for the gastric mucosa.



In emphysema, the destruction of many alveolar walls changes the compliance of the respiratory system. Which
of the following clinical observations is directly related to this change in compliance?


A. Barrel chest

B. Chronic cough

C. Excessive mucus production

D. Long, slow, deep breathing pattern

E. Pink face


Explanation:

The correct answer is A. A barrel chest with increased anterior/posterior diameter is commonly observed in
patients with long-standing, severe emphysema. This change in chest shape occurs because these patients,
who have high compliance of the lung proper, tend to function with their lungs to some degree "over-inflated"
compared to people with normal lung compliance. This over-inflation limits their ability to take further deep
breaths. (The "balloon" of emphysematous lung remains compliant, but the "box" of the chest wall is not very
compliant and limits the volume of air that can be inhaled). Patients with moderately severe emphysema are
able to maintain an adequate lung ventilation by taking many short breaths (compare with choice D); this
physiology is sometimes expressed by describing these patients as "pink puffers" (choice E).

Chronic cough (choice B) in emphysema patients is not directly related to the change in compliance.

Excessive mucus production (choice C) is more characteristic of chronic bronchitis than of emphysema.



A 54-year-old alcoholic presents with complaints of tremors and muscle twitching. Physical examination reveals
the presence of Trousseau's sign. Laboratory data show that serum magnesium is < 1 mEq/L (normal, 1.4 - 2.2
mEq/L). Which of the following findings would be most consistent with this information?


A. Decreased serum calcium

B. Decreased serum phosphate

C. Increased bone density

D. Increased plasma parathyroid hormone concentration

E. Increased urinary cAMP concentration


Explanation:

The correct answer is A. Malnutrition associated with chronic alcoholism can lead to a severe magnesium
deficiency. The effect of low serum magnesium on parathyroid hormone secretion (PTH) depends on severity
and duration. An acute decrease in serum magnesium will increase PTH secretion, while a prolonged severe
deficiency results in decreased PTH secretion. There is also evidence that the action of PTH is decreased with
chronic magnesium deficiency. Hence, this patient is suffering from "functional" hypoparathyroidism. The low
serum calcium can produce weakness, tremors, muscle fasciculations, and seizures. A positive Trousseau's
sign indicates the presence of latent tetany. It is observed by inflating a blood pressure cuff above systolic
blood pressure for at least 2 minutes. A positive reaction consists of the development of carpal spasm, with
relaxation occurring within seconds after deflating the cuff. In patients with magnesium deficiency, magnesium
administration will produce a prompt rise in plasma PTH with subsequent restoration of serum calcium
concentration to normal.

With functional hypoparathyroidism bone density would be decreased (not increased, choice C).

The combination of decreased PTH secretion (not increased, choice D) and decreased effectiveness of PTH
produce hypocalcemia and hyperphosphatemia (not hypophosphatemia, choice B).

Urinary cAMP would probably be decreased (not increased, choice E), given the low PTH.



A child who has had abnormal development of the membranous bones has a broad skull with associated facial
and dental anomalies. Which other bones are most likely to also be affected?


A. Clavicles

B. Femurs

C. Metatarsals

D. Phalanges

E. Tibias


Explanation:

The correct answer is A. In a syndrome called cleidocranial dysostosis, absence of part of the clavicles
accompanies a broad skull, and facial and dental anomalies. Note that you could also have answered this
question by noting that of the bones listed, only the clavicles form by intramembranous ossification.

The femurs (choice B), metatarsals (choice C), phalanges (choice D), and tibias (choice E) are cartilaginous
(formed by endochondral ossification) rather than membranous bones.



A 50-year-old man presents to his doctor with diarrhea, flushing, and wheezing. Physical examination is
significant for a grade II/VI diastolic murmur located at the right sternal border at the 4th intercostal space. Which
of the following substances is most likely to be elevated in this patient's urine?


A. 5-HIAA

B. HVA

C. Phenylalanine

D. Selegiline

E. Vanillylmandelic acid (VMA)


Explanation:

The correct answer is A. 5-HIAA is a metabolite of serotonin, a major secretory product of carcinoid tumors. The
signs and symptoms of carcinoid syndrome include diarrhea, flushing, and wheezing. The cardiac abnormalities
are commonly concentrated in the right heart because carcinoid secretory products are degraded or detoxified
in the lung.

HVA (choice B) is a breakdown product of dopamine through the MAO or COMT metabolism pathways.

Phenylalanine (choice C) is an essential amino acid that is used to synthesize tyrosine, the precursor of the
catecholamines (dopamine, norepinephrine, and epinephrine).

Selegiline (choice D) is a MAO-B inhibitor that inhibits the degradation of dopamine. It is used in the treatment
of Parkinson's disease.

VMA (choice E) is a metabolite of epinephrine that is elevated in the urine of individuals with
pheochromocytoma.


A 26-year-old male is brought to a physician because of a head injury. His wife states that she and her husband
were walking on the sidewalk when he suddenly fell to the ground and hit his head. She said similar episodes had
occurred before, but this was the first time that he had been injured. Cardiovascular evaluation is unrevealing. An
electroencephalogram administered at the appropriate time would probably reveal which of the following types of
seizures?


A. Absence

B. Atonic

C. Myoclonic

D. Tonic

E. Tonic-clonic


Explanation:

The correct answer is B. Atonic or "drop" seizures are characterized by a sudden loss of postural muscle tone
that lasts only a few seconds. Although consciousness may be impaired briefly, there is rarely postictal
confusion. A very brief seizure may cause only a drop of the head, but a longer seizure may cause the patient
to slump to the ground. This type of seizure may be quite dangerous because of the risk of head injury with a
sudden fall. Drugs prescribed for this condition include valproic acid, clonazepam, felbamate, vigabatrin, and
lamotrigine.

Absence seizures (choice A), also known as petit mal seizures, are characterized by blank stares and an
absence of any change in position. They typically occur in children. Drugs used in this disorder include
ethosuximide, valproic acid, and clonazepam.

Myoclonic seizures (choice C) are characterized by sudden, brief muscle jerks that may involve part of the body
or the whole body. A physiologic form of myoclonus occurring in healthy individuals is the sudden jerking
movement that sometimes occurs while falling asleep. Drugs used for this condition include valproic acid and
clonazepam.

Tonic seizures (choice D) are characterized by the sudden onset of sustained axial and limb muscle contraction.
Pure tonic seizures tend to occur in children.

Tonic-clonic seizures (choice E), also known as grand mal seizures, are characterized by an initial phase with
tonic contraction of muscles throughout the body. After a short time, the seizure evolves into the clonic phase,
in which periods of muscle relaxation are superimposed on muscle contraction. There is a significant postictal
phase. Drugs used for this disorder include carbamazepine, phenytoin, and valproic acid.




A cyanotic infant is discovered to have a ventricular septal defect, an overriding aorta, right ventricular
hypertrophy, and complete pulmonic stenosis. Which of the following accompanying congenital anomalies
permits survival in this patient?


A. Bicuspid aortic valve

B. Ostium secundum defect

C. Patent ductus arteriosus

D. Patent foramen ovale

E. Preductal coarctation of aorta


Explanation:

The correct answer is C. The ductus arteriosus connects the aorta with the pulmonary artery. If it remains
patent after birth, it allows oxygenated blood to flow from the aorta to the pulmonary artery. In this patient with
tetralogy of Fallot with complete right ventricular outflow obstruction, this anastomosis is a crucial source of
blood to the pulmonary vasculature.

A bicuspid aortic valve (choice A) may be asymptomatic but can lead to infective endocarditis, left ventricular
overload, and sudden death. It is a common cause of aortic stenosis. It would not benefit a patient with
tetralogy of Fallot in any way.

Ostium secundum defect (choice B) is the most common form of atrial septal defect (ASD). ASD is an acyanotic
congenital heart disease that would not improve cardiovascular function in a patient with tetralogy of Fallot.

A patent foramen ovale (choice D) is a slit-like remnant of communication between the left and right atria in the
fetus. It is usually not of clinical significance.

A preductal coarctation of the aorta (choice E) involves narrowing of the aorta proximal to the opening of the
ductus arteriosus. This would prevent adequate blood flow through a possible life-preserving PDA and would
result in the patient's demise.



A 14-year-old male presents with type I diabetes mellitus. His mother wants to know if the boy's brother might also
have an increased risk of getting the disease. Which of the following genotypes, if present in the brother, would
be associated with the greatest risk of developing diabetes?


A. B27/B27

B. DR2/DR2

C. DR2/DR4

D. DR3/DR3

E. DR3/DR4


Explanation:

The correct answer is E. A heterozygous individual with HLA-DR3 and HLA-DR4 has a 33-fold greater relative
risk for developing diabetes than individuals without these two HLA antigens.

Homozygous individuals with HLA-B27 (choice A) are more likely to develop ankylosing spondylitis and Reiter's
disease.

Patients who are homozygous for HLA-DR2 (choice B) have a reduced risk of diabetes mellitus. Individuals with
the HLA-DR4 allele have an increased risk of diabetes. Heterozygotes for HLA-DR2 and HLA-DR4(choice C)
likely have an intermediate risk.

Individuals who are positive for the HLA-DR3 antigen (choice D) have three times the risk of developing
diabetes mellitus.



A patient hospitalized with pneumonia has a thyroid hormone panel ordered along with other routine blood work.
Serum T3 is decreased, but serum T4 and TSH are within the normal range. From this information, the physician
concludes that the patient


A. has low T3 syndrome (euthyroid sick syndrome)

B. has primary hypothyroidism

C. should be treated with thyroxine replacement

D. will also exhibit decreased serum reverse T3 concentration

E. will also exhibit increased activity of 5'-monodeiodinase in peripheral tissues


Explanation:

The correct answer is A. The low T3 syndrome or "euthyroid sick syndrome" occurs with certain systemic
illnesses like pneumonia or septicemia, after major surgery, and with malnutrition or starvation. Whatever the
cause, the syndrome is characterized by a decrease, not increase (choice E), in 5'-monodeiodinase activity in
peripheral tissues like liver and kidney. As a consequence the conversion of circulating T4 to T3 is impaired and
blood levels of T3 decrease. The decrease in serum T3 is thought to be a protective adaptation to decrease
catabolic processes during the illness or shortage of energy substrates.

Primary hypothyroidism (choice B) is characterized by decreased serum T4 and increased serum TSH. In the
euthyroid sick syndrome, serum T4 and TSH are usually within the normal range.

Because the low T3 is probably a protective adaptation, treatment with thyroxine replacement (choice C) is of
little benefit and may actually be harmful. Once the patient recovers from the illness or malnutrition, serum
levels of T3 gradually return to normal on their own.

A normal step in the breakdown of reverse T3 also involves the action of 5'-monodeiodinase, which converts
reverse T3 to 3,3'-diiodothyronine. Hence, with the low T3 syndrome, serum concentration of reverse T3 is
increased, not decreased (choice D), due to decreased breakdown.



A 48-year-old male is seen for persistent edema. Initial laboratory studies indicate that he has nephrotic
syndrome, and a renal biopsy is diagnostic of membranous glomerulonephritis. Which of the following substances
will be elevated in the plasma in this patient?


A. Albumin

B. Ammonia

C. Cholesterol

D. Glucose

E. Potassium


Explanation:

The correct answer is C. The nephrotic syndrome describes a group of laboratory findings associated with
glomerular diseases which share the common characteristic of "leaky glomeruli." Large biochemicals, normally
unable to cross out of glomerular capillaries into Bowman's space, are lost into the urine. Serum proteins are
lost in large quantities and can be detected as both hypoproteinemia and massive proteinuria. Albumin (choice
A), a relatively small plasma protein (MW 66,000) is lost very readily, leading to hypoalbuminemia. Serum
concentrations of small compounds such as potassium (choice E) and glucose (choice D), which are highly
permeable in the normal glomerulus, are unaffected by glomerulonephritides producing the nephrotic
syndrome. Similarly, blood urea nitrogen is unaffected, and serum ammonia levels (choice B) are unchanged.
The final component of the nephrotic syndrome (besides hypoproteinemia, hypoalbuminemia and massive
proteinuria) is hyperlipidemia. It is apparently a function of both increased hepatic fat synthesis and decreased
fat catabolism. Increased cholesterol (choice C), triglycerides, and lipoproteins are found in serum in
membranous glomerulonephritis, and these lipids leak into the urine, producing lipiduria.



A patient with mild congestive heart failure is treated with high-dose furosemide and diureses 25 pounds of fluid.
A complete blood count (CBC) taken before the diuresis shows an RBC count of 4 million/mm3; a CBC taken
after diuresis shows a RBC count of 7 million/mm3. Which of the following is the most likely explanation?


A. Cyanotic heart disease

B. Increased erythropoietin

C. Polycythemia vera

D. Relative polycythemia

E. Renal cell carcinoma


Explanation:

The correct answer is D. This is an example of relative polycythemia, in which there is an increased hematocrit
or RBC count without a true increase in the total number of body RBCs. What usually happens in these cases
is a significant reduction in plasma volume due to processes such as dehydration, vomiting, diarrhea, or
diuresis.

Cyanotic heart disease (choice A), via appropriate erythropoietin secretion, can cause secondary absolute
polycythemia.

Increased erythropoietin (choice B), whether appropriately or inappropriately secreted, can cause secondary
absolute polycythemia.

Polycythemia vera (choice C) causes primary absolute polycythemia with usually low erythropoietin levels.

Renal cell carcinoma (choice E), via inappropriate erythropoietin secretion, can cause secondary absolute
polycythemia.



An elderly female with a history of alcoholic disease develops jaundice and marked anasarca. Which of the
following is the most likely pathophysiology of her persistent edema?


A. Lymphatic obstruction

B. Reduced central venous pressure

C. Reduced plasma oncotic pressure

D. Sodium retention

E. Venous thrombosis


Explanation:

The correct answer is C. Hepatic failure occurring in cirrhosis reduces the capacity of the liver to synthesize
sufficient quantities of plasma proteins (mostly albumin) necessary to maintain plasma oncotic pressure. Low
plasma oncotic pressure allows fluid from the intravascular fluid component to move into the interstitial space,
producing plasma volume contraction and edema.

Lymphatic obstruction (choice A) occurs as a result of mechanical blockage of lymphatics by tumor,
inflammatory processes, or certain parasitic infections. Cirrhosis does not lead to lymphatic obstruction.

Reduced central venous pressure (choice B) does not cause edema. Conversely, increased central venous
pressure, which may arise with congestive heart failure, thrombosis, or cirrhosis can lead to increased
hydrostatic pressure and edema.

Sodium retention (choice D) is an important cause of edema in patients with poor renal perfusion. The kidneys
respond by retaining sodium and increasing plasma volume in an effort to increase renal blood flow. Sodium
retention in cirrhosis is secondary to the decrease in plasma oncotic pressure and consequent decrease in
plasma volume.

Venous thrombosis (choice E) can lead to edema; however, the diminished synthesis of coagulation proteins in
cirrhosis predisposes to bleeding, not thrombosis.



Physical examination of a neonate is remarkable for a holosystolic murmur. There is no cyanosis.
Echocardiography demonstrates an ostium primum defect in the lower part of the interatrial septum that is
accompanied by malformations of the adjacent atrioventricular valves. These lesions are most likely associated
with which of the following disorders?


A. Cystic fibrosis

B. Down syndrome

C. Gaucher disease

D. Marfan syndrome

E. Turner syndrome


Explanation:

The correct answer is B. The most common type of atrial septal defect is the ostium secundum type. Children
with Down syndrome, however, are frequently afflicted with the ostium primum type of atrial septal defects,
which may be accompanied by tricuspid and mitral valve malformations. More complex atrioventricular septal
defects may also occur in this disorder. Children exhibiting these lesions should be specifically evaluated for
chromosomal abnormalities. Clinically, the lesions produce left-to-right shunts with late cyanosis (after the right
ventricle hypertrophies in response to developing lung disease from the increased blood flow in the pulmonary
system).

Neither cystic fibrosis (choice A) nor Gaucher disease (choice C) is specifically associated with cardiovascular
defects.

Dissecting aortic aneurysm is associated with Marfan syndrome (choice D).

Turner syndrome (choice E) is associated with coarctation of the aorta.



An 18-year-old male comes to the university clinic supported by his roommates because he cannot walk. He
describes a rapidly evolving weakness affecting his legs and feet starting 2 days ago. On physical examination he
cannot move his feet or ankles and he can barely raise his thighs off the bed. He has symmetrical hyporeflexia of
the legs, but his sensorium is completely intact. Scanning his chart, the physician notes that he was treated 10
days previously for an upper respiratory tract infection. The immunological response producing the patient's
symptoms is most intense at which of the following locations?


A. Lateral corticospinal tracts

B. Neuromuscular junction

C. Precentral gyrus

D. Skeletal muscles

E. Spinal motor nerves


Explanation:

The correct answer is E. The patient has developed Guillain-Barré syndrome, also known as inflammatory
polyneuropathy. This presentation is classic-rapidly evolving limb weakness with symmetrical hyporeflexia but
normal sensation. The syndrome frequently follows viral infections and may evolve into complete paralysis with
respiratory failure. Guillain-Barré syndrome is thought to be an autoimmune disease. The clinical course is
correlated with a chronic inflammatory infiltrate and demyelination of peripheral nerves, especially spinal and
cranial motor nerve roots.

Inflammation localized to a small portion of spinal cord (lateral corticospinal tracts; choice A) or cerebral cortex
(precentral gyrus; choice C) may occur in progressive multifocal leukoencephalopathy (PML) or in multiple
sclerosis (MS). PML occurs in the immunosuppressed, and MS presents with hyperreflexia (upper motor neuron
signs).

The classic autoimmune disease involving the neuromuscular junction (choice B) is myasthenia gravis. Although
the weakness caused by myasthenia gravis may affect the legs, extraocular muscles are involved in the majority
of cases, and isolated limb weakness is rare.

Diseases primarily affecting the skeletal muscle (choice D) include autoimmune inflammatory myopathies such
as dermatomyositis and polymyositis, which typically affect the proximal limb muscles more than the distal
musculature. Inclusion body myositis is a slowly developing disease that is asymmetrical and occurs in older
individuals.


A 74-year-old woman with type II diabetes mellitus, hypertension, and end-stage renal failure has been
dialysis-dependent for several years. She develops a fracture of the left femoral head. Tissue taken from the
fracture at the time of internal fixation is noted to have increased osteoclastic activity, with notable tunnel-like
dissection by osteoclasts into the bony trabeculae. Increased levels of which of the following hormones is most
likely to be responsible for this lesion?


A. Calcitonin

B. Cortisol

C. Erythropoietin

D. Glucagon

E. Parathyroid hormone


Explanation:

The correct answer is E. The patient has developed hyperparathyroidism, a well-recognized sequela of chronic
renal failure. This condition is caused by high levels of serum phosphate and low serum calcium, which
stimulate the release of parathyroid hormone (PTH) in an effort to normalize the calcium/phosphate ratio. PTH
stimulates osteoblasts to become osteoclasts, which dissolve the bone reservoir of calcium and release it into
the blood. This condition, known as renal osteodystrophy, may lead to osteomalacia and osteitis fibrosa cystica,
which is classically associated with dissecting osteitis, as described in this question.

Calcitonin (choice A), which serves to lower serum calcium levels, is produced in small quantities in chronic
renal failure, as serum calcium is already pathologically low.

Excess cortisol (choice B; Cushing's syndrome) may produce osteoporosis and pathologic fractures, but there
is no relationship between chronic renal failure and cortisol excess. Cushing's syndrome is generally secondary
to adrenal or pituitary adenomas or primary adrenal hyperfunction.

Erythropoietin (choice C) production is deficient in chronic renal failure. Excess erythropoietin activates
erythrocyte precursors, but does not activate osteoclasts.

Glucagon (choice D) excess is a very rare entity, occurring in a minority of islet cell tumors, and is not
associated with renal failure. High glucagon produces a transitory skin rash, anemia, and a form of diabetes
mellitus.


A 25-year-old male reports episodic "spells" characterized by palpitations, sweating, nervousness, and feelings of
anxiety. On examination, the man's blood pressure is 165/95 mm Hg. Plasma norepinephrine is 450 pg/mL
(normal, 150-400 pg/mL), plasma epinephrine is 115 pg/mL (normal, 25-100 pg/mL), and 24-hour urinary VMA is
11 mg (normal, < 8 mg). Which of the following is the most likely cause of the patient's hypertension?


A. 11-beta-hydroxylase deficiency

B. Conn's syndrome

C. Pheochromocytoma

D. Renin-secreting tumor

E. Unilateral renal artery stenosis


Explanation:

The correct answer is C. A pheochromocytoma is a tumor arising from chromaffin cells that secretes excess
catecholamines (norepinephrine, epinephrine, or both). It is one endocrine cause of hypertension, due to
peripheral vasoconstriction and/or increased cardiac output. While most patients have higher than normal
baseline plasma levels of catecholamines, it is not uncommon for paroxysmal symptomatic episodes to be
superimposed upon the basal problem. These "attacks" may occur several times a week (or more often) and
last for up to 15 minutes. During an attack, respiration can increase, the patient may become aware of a forceful
pounding of the heart that progresses to include a throbbing headache, and peripheral vasoconstriction can
raise body temperature and lead to reflex sweating. Marked anxiety may also accompany the episode.
Diagnosis can be confirmed by measuring increased plasma or urinary catecholamines or their metabolites.

11-beta-hydroxylase deficiency (choice A) is a congenital disorder than can cause hypertension due to
excessive production of the weak mineralocorticoid, deoxycorticosterone, by the inner two zones of the adrenal
cortex. This results in excessive renal retention of sodium and water and subsequent hypertension. While the
hypertension is usually present from birth, a late-onset variant of this disorder has been described in which the
symptoms do not present until late childhood or adolescence. Virilization is also present due to excessive
secretion of adrenal androgens.

Conn's syndrome (choice B) is another endocrine cause of hypertension. In this case, the increase in blood
pressure is due to excessive renal retention of sodium and water resulting from the increased plasma
concentration of aldosterone. The paroxysmal symptoms and increased catecholamines present in this patient
are not present with primary hyperaldosteronism.

Renin-secreting tumors (choice D) are rare and can be confused with primary hyperaldosteronism. The
excessive secretion of renin by the tumor can increase the formation of angiotensin II with subsequent
hyperaldosteronism. Sodium and water retention, together with hypokalemia, are present. Increased plasma
renin and plasma aldosterone are suggestive of a renin-secreting tumor, whereas primary hyperaldosteronism
would present as increased plasma aldosterone, but decreased plasma renin.

Unilateral renal artery stenosis (choice E) produces an angiotensin II-dependent form of hypertension.
Decreased renal perfusion, often due to atherosclerosis or fibromuscular hyperplasia of the renal arteries,
results in increased renin secretion and subsequently increased plasma angiotensin II. The resultant increase in
aldosterone secretion and arteriolar vasoconstriction contributes to the hypertension.





A 74-year-old woman, in otherwise good health, tripped and injured her right leg 2 days previously and has been
bedridden since the accident. Two hours ago, she became delirious. On physical examination, her temperature is
99 F, blood pressure is 120/70 mm Hg, heart rate is 110, and respiratory rate is 32. Pulse oximetry shows an
oxygen saturation of 80%, and a chest x-ray film is normal. Which of the following is the most likely diagnosis?


A. Acute cerebral hemorrhage

B. Acute cerebral infarction

C. Myocardial infarction

D. Pulmonary infarction

E. Pulmonary thromboembolism


Explanation:

The correct answer is E. Hip fracture and prolonged bed rest are classic risk factors for the development of
pulmonary thromboemboli (PE). Common clinical manifestations of PE are hypoxia (due to ventilation/perfusion
mismatch) despite a normal chest x-ray, tachycardia, and delirium in older patients.

Cerebral hemorrhage (choice A) might cause delirium but would not directly cause hypoxia unless the patient
was hypoventilating (e.g., because of brainstem involvement).

Cerebral infarction (choice B) could produce delirium but would not directly cause hypoxia unless the patient
was hypoventilating.

Myocardial infarction (choice C) could account for delirium and tachycardia, but not for hypoxia with a normal
chest x-ray. Severe congestive heart failure after myocardial infarction could cause hypoxia due to pulmonary
edema, but the chest x-ray would not be normal.

Pulmonary infarction (choice D) may cause delirium, tachycardia, and hypoxia, but the chest x-ray may be
abnormal. A chest x-ray performed within 12-36 hours after a pulmonary infarct may reveal a peripherally
located, wedge-shaped infiltrate.



A 26-year-old woman with sickle cell anemia develops pneumonia, and as a complication, develops very painful
focal ischemia in the muscles and joints. Which part of the renal vasculature would be most vulnerable to similar
damage during this sickle cell crisis?


A. Arcuate arteries

B. Glomerular capillaries

C. Interlobular arteries

D. Renal artery

E. Vasa recta


Explanation:

The correct answer is E. Sickling crises can be triggered by hypoxia caused by high altitude or pneumonia. The
very high osmolarity of the renal medulla particularly favors sickling of erythrocytes in the vasa recta. The
resultant ischemia can cause a patchy papillary necrosis, proteinuria, and sometimes, cortical scarring.

Larger blood vessels such as the arcuate arteries (choice A), interlobar arteries (choice C), and renal arteries
(choice D) are usually not occluded by sickled cells.

The glomerular capillaries (choice B) are small enough to be occluded by sickled cells, but they are well
oxygenated and the blood within them is not hypertonic, so the glomerular capillaries are significantly less likely
to become occluded then the vasa recta.



In retinitis pigmentosa, "night-blindness" is an early symptom of visual loss. Which of the following explains this
phenomenon?


A. Cones are relatively preserved compared to ganglion cells

B. Cones are relatively preserved compared to rods

C. Ganglion cells are relatively preserved compared to cones

D. Ganglion cells are relatively preserved compared to rods

E. Rods are relatively preserved compared to cones


Explanation:

The correct answer is B. Retinitis pigmentosa is a familial degenerative disease of the retina that most often has
recessive genetics. The initial problem appears to be alterations in the pigmented epithelium below the retina,
particularly in the most anterior portions of the retina. The pigmented epithelial cells become disrupted and leak
pigment, which accumulates along the attenuated blood vessels (and can be seen with an ophthalmoscope). A
consequence of this damage is that the rod cells that are normally nutritionally supported and "groomed" by the
pigment epithelium also undergo degenerative changes. Since the cones are relatively preserved, day vision is
preserved, but night vision, which is highly dependent on rods, begins to decay. With disease progression,
vision is completely or nearly completely lost and the retina becomes so distorted that only a single row of cone
nuclei with scattered stumpy cone remnants is all that remains of the photoreceptor layer.



A 32-year old women complains of amenorrhea since delivery of a baby 15 months previously, despite the fact
that she did not breast feed her baby. The delivery was complicated by excessive hemorrhage that required
transfusion of 2.5 liters of blood. She has also been fatigued and has gained an additional 10 pounds since the
baby was born. Laboratory data show the following:

Serum LH < 1 IU/L (normal, 4-24 IU/L)

Serum estradiol 5 pg/mL (normal, 20 - 100 pg/mL)

Serum TSH 0.1 mU/L (normal, 0.5 - 5 mU/L)

Serum GH 3 ng/mL (normal, < 5 ng/mL)

Serum ACTH 28 pg/mL (normal, 10 - 50 pg/mL)

Serum prolactin 2 ng/mL (normal, <20 ng/mL)

Injection of 500 µg of TRH failed to produce the expected rise in both serum TSH and prolactin. Which of the
following diagnoses most likely explains the findings in this patient?


A. Hashimoto's thyroiditis

B. Isolated gonadotropin deficiency

C. Primary amenorrhea

D. Prolactinoma

E. Sheehan's syndrome


Explanation:

The correct answer is E. Sheehan's syndrome is hypopituitarism due to ischemic damage to the pituitary
resulting from excessive hemorrhage during parturition. The pituitary is enlarged during pregnancy; it is more
metabolically active, and more susceptible to hypoxemia. Furthermore, the blood vessels in the pituitary may be
more susceptible to vasospasm because of the high estrogen. In about 30% of women who hemorrhage
excessively during parturition, some degree of hypopituitarism eventually becomes manifest. The symptoms
depend on how much of the pituitary is damaged and what cell types are destroyed. The patient described
above exhibited persistent amenorrhea after delivery of her infant. This is due to destruction of pituitary
gonadotrophs and diminished secretion of gonadotropins (LH). There also appears to have been significant
destruction of lactotrophs since TRH injection failed to induce an increase in prolactin. Had the women
attempted to breast-feed her infant, a failure to lactate mostly likely would have occurred. This case is also
characterized by secondary hypothyroidism. The low TSH and failure to respond to TRH injection is
confirmatory. Corticotrophs appear to have been spared since plasma ACTH is normal. It is not clear whether
somatotrophs were damaged. Further testing would be needed to see if GH reserve is diminished.

Hashimoto's thyroiditis (choice A) is an autoimmune disorder that produces primary hypothyroidism. Because of
diminished negative feedback effects of T4, serum TSH is usually increased (not decreased).

Isolated gonadotropin deficiency (choice B) produces amenorrhea and is associated with low serum LH and
estradiol. Hypogonadotropic hypogonadism can occur in female athletes that over-train, in anorexia nervosa, in
obesity, or with other emotional or physical stresses. However, other pituitary hormones are unaffected (by
definition isolated gonadotropin deficiency only involves a decrease in gonadotropins).

Primary amenorrhea (choice C), by definition, means failure of menstrual cycles to ever begin. Since this woman
has delivered a baby, primary amenorrhea is highly unlikely.

A prolactinoma (choice D) is a functional pituitary tumor that secretes excessive prolactin. This can cause
amenorrhea by suppressing the GnRH-pituitary-gonad axis. The patient described above has decreased
prolactin secretion.



A 40-year-old man presents to his physician with complaints of substernal pain radiating to his left shoulder,
occurring when he is at rest. The pain improves when he gets up and moves around, and responds to sublingual
nitroglycerin. This patient's symptoms are thought to be caused by which of the following processes?


A. Coronary artery atherosclerosis

B. Coronary artery embolism

C. Coronary artery spasm

D. Coronary artery thrombosis

E. Coronary artery vasculitis


Explanation:

The correct answer is C. The pattern of angina described is called Prinzmetal's variant angina, and is believed
to be due to coronary artery spasm occurring during rest. Up to 75% of patients with this disorder can be shown
to have a fixed obstruction within 1 cm of the site of arterial spasm. Patients with Prinzmetal's angina are often
younger than patients with unstable angina secondary to coronary atherosclerosis. The diagnosis may be
confirmed by observing a spontaneous coronary artery spasm (or provoking one by administering ergonovine
or acetylcholine, or instructing the patient to hyperventilate) during angiography.

Atherosclerosis (choice A) is the cause of typical angina that occurs with exertion.

Embolism (choice B) in the coronary arteries is somewhat unusual, but can occur if a plaque from the aorta
breaks off and lodges in a coronary vessel.

Thrombosis (choice D) characteristically produces unstable or crescendo angina with worsening chest pain.

Vasculitis (choice E) can uncommonly cause angina, notably as part of Kawasaki's disease.



A 21-year-old woman attempted suicide by taking an overdose of barbiturates. On arrival in the emergency
department, her blood pressure is 95/65 and her pulse is 105 per minute. The physician in the intensive care
unit orders arterial blood gases. Which of the following values would you expect in this patient?


A. PO2 = 45, PCO2 = 45, pH = 7.45

B. PO2 = 55, PCO2 = 70, pH = 7.50

C. PO2 = 65, PCO2 = 35, pH = 7.45

D. PO2 = 75, PCO2 = 60, pH = 7.30

E. PO2 = 98, PCO2 = 60, pH = 7.20


Explanation:

The correct answer is D. Barbiturate overdose causes respiratory depression, resulting in carbon dioxide
retention (producing increased PCO2 and decreased pH) and hypoxemia (decreased PO2). In other words, the
patient has respiratory acidosis. You should look for a low PO2, high PCO2, and acidotic pH. Choices A, B, C,
and E do not fulfill these requirements. Note that choice C might be expected in a patient who is
hyperventilating to the point of respiratory alkalosis: diminished O2 (the usual drive for hyperventilation in
nonpsychiatric hyperventilation), diminished CO2, and mildly alkalotic pH.



A baby is born prematurely at 28 weeks gestational age with a birthweight of 1200 g. Ventilation with high partial
pressures of oxygen is started, but the neonatologist is concerned about the possible development of retinopathy
of prematurity (formerly called retrolental fibroplasia). What is the underlying mechanism by which this retinal
lesion may develop?


A. Ganglion cell degeneration

B. Inappropriate vascular proliferation

C. Optic nerve degeneration

D. Pigment deposition in retina

E. Pigmented epithelium degeneration


Explanation:

The correct answer is B. The underlying lesion in retinopathy of prematurity (retrolental fibroplasia) is an
inappropriate proliferation of vessels in the inner layers of the retina. If the process is disrupted early, the retina
may heal with little residual damage. However, persistence of the process predisposes for permanent damage
due to exudation, hemorrhage, and secondary detachment of the retina.

Ganglion cell and optic nerve degeneration (choices A and C) are features of retinal damage due to glaucoma.

Pigment deposition in the retina (choice D) is unrelated to retinopathy of prematurity.

Pigmented epithelium degeneration (choice E) is a feature of retinitis pigmentosa and also of senile macular
degeneration.



A 45-year-old male complains of fatigue and increased frequency of urination. Questioning reveals that he is
somewhat confused. Physical examination reveals a 5'10", 240 lb. individual whose fat is centrally distributed.
Urine is positive for glucose, but negative for ketones. A blood sample drawn from this patient is likely to exhibit
which of the following compared to that of a normal individual?


A. Decreased concentration of C-peptide

B. Decreased pH

C. Increased antibodies against islet cell proteins

D. Increased concentration of β-hydroxybutyrate

E. Increased osmolarity


Explanation:

The correct answer is E. An obese adult with glucosuria, but not ketonuria, likely has type 2 diabetes mellitus.
Type 2 diabetes is characterized by insulin resistance resulting in hyperglycemia and increased serum
osmolarity. The dehydration associated with osmotic diuresis makes the hyperosmolarity worse. As the
osmolarity increases above 330 mOsm/L, the osmotic loss of water from neurons is sufficient to produce coma.
In nonketotic, hyperosmolar coma, blood glucose values can range from 800 to 2,400 mg/dL and produce
serum osmolarities of 330-440 mOsm/L.

Since type 2 diabetes is due to insulin resistance, plasma levels of insulin are usually normal to increased.
Because β cells secrete insulin and C-peptide in a 1:1 ratio, plasma concentration of C-peptide would also
be normal to increased (not decreased, choice A) in type 2 diabetes.

Even small amounts of insulin are sufficient to prevent ketosis. In type 2 diabetes there is enough insulin effect
to prevent significant lipolysis and subsequent formation of excess ketone bodies. Hence, acidosis (choice B) is
not typically associated with this disorder. Since β-hydroxybutyrate is a ketone body, its concentration in
plasma is not likely to be increased (choice D).

While there is a significant autoimmune component to type 1 diabetes, type 2 diabetes is not associated with
increased circulating antibodies (choice C) against β cell proteins such as glutamic acid decarboxylase.



A 7-year-old child is brought to the pediatrician because of increased thirst and bed-wetting. Despite an
increased appetite, she has lost 5 pounds over the past month. A dipstick test of a urine sample reveals the
presence of glucose. A blood sample drawn from this patient is most likely to exhibit which of the following
compared to that of a normal individual?


A. Decreased concentration of β-hydroxybutyrate

B. Decreased concentration of hemoglobin A1c

C. Decreased osmolarity

D. Increased antibodies against glutamic acid decarboxylase (GAD)

E. Increased concentration of C-peptide


Explanation:

The correct answer is D. A child with glucosuria, polyuria, polydipsia, and weight loss despite increased appetite
is likely to have type 1 diabetes mellitus. This form of diabetes has an important autoimmune component to its
etiology. As many as 85% of patients have islet cell autoantibodies in their blood in the first few weeks after
onset of the disease. Most of these antibodies are directed against glutamic acid decarboxylase, an enzyme that
is present in β cells.

The insulin deficiency associated with autoimmune destruction of the β cells leads to increased breakdown
of triglycerides with a consequently increased rate of formation of ketone bodies. The primary ketone body
produced is β-hydroxybutyrate (choice A), the blood level of which is increased (not decreased) in type 1
diabetes. If untreated, type 1 diabetes can rapidly progress to ketoacidosis and coma.

Insulin deficiency leads to decreased glucose uptake and hyperglycemia. The increase in blood glucose results
in increased glycosylation of hemoglobin in red blood cells. Hence, the hemoglobin A1c concentration will be
increased, not decreased (choice B), reflecting long-standing hyperglycemia.

Another consequence of the increased blood glucose is increased serum osmolarity (not decreased, choice C).
If the osmolarity exceeds 330 mOsm/L, consciousness begins to be impaired.

β cell destruction in type 1 diabetes leads to decreased secretion of insulin. Since insulin and C-peptide
are released on a 1:1 ratio by the β cells, the secretion of C-peptide is also decreased (not increased,
choice E). Insulin is first synthesized as preproinsulin, with an initial leader sequence of hydrophobic amino acids
that allows the developing protein to be extruded from the ribosomes into the rough endoplasmic reticulum. The
"pre" segment is then cleaved off, leaving proinsulin. Proinsulin is packaged into secretory granules within the
Golgi, and as the granules bud off, enzymes cleave the precursor into insulin and C-peptide.



A 67-year-old man presents to his doctor's office for an insurance physical. During blood pressure measurement,
the nurse notes that systolic sounds are heard with the cuff completely deflated. The blood pressure is 180/60
mm Hg. Physical examination reveals bounding pulses and a high-pitched, blowing diastolic murmur, heard best
along the left sternal border. Which of the following is the most likely diagnosis?


A. Aortic regurgitation

B. Aortic valve obstruction

C. Cardiac tamponade

D. Heart failure

E. Hypovolemia


Explanation:

The correct answer is A. The case described is typical for aortic regurgitation. In aortic regurgitation, blood
flows backward through the aortic valve during diastole when the valve is closed. The arterial pulse pressure is
widened, often to over 100 mm Hg (normal pulse pressure is 30 to 50 mm Hg), and in extreme cases the
systolic pressure can be elevated to over 250 mm Hg with a depression in the diastolic pressure. Note that the
pulse pressure is 180-60 = 120 mm Hg in this case. The aortic pressure falls greatly during diastole because
blood from the aorta regurgitates backward into the ventricle through the leaky aortic valve. The increase in
systolic pressure most likely results from the large increase in stroke volume, which is secondary to an increase
in the end diastolic volume. The diastolic murmur of aortic regurgitation is typically a high-pitched, blowing,
decrescendo murmur, best heard along the left sternal border over the third intercostal space.

The pulse pressure is decreased in aortic valve obstruction (choice B), cardiac tamponade (choice C), heart
failure (choice D), and hypovolemia (choice E).


A 46-year-old woman presents to her doctor complaining of weakness and fatigue. On physical examination, her
physician notices a 10-pound weight gain since her last visit 6 months ago. Her blood pressure is 160/100 mm
Hg. Blood tests reveal serum Na+ 155 mEq/L, K+ 2.8 mEq/L, and a decreased serum renin. Which of the following
is the most likely diagnosis?


A. Cushing's syndrome

B. Diabetes mellitus

C. Pheochromocytoma

D. Primary aldosteronism

E. Secondary aldosteronism


Explanation:

The correct answer is D. Primary aldosteronism (Conn's syndrome) is a condition of hyperaldosteronism
originating in the adrenal gland. The causes include an aldosterone-secreting adrenocortical adenoma,
hyperplasia of the zona glomerulosa, and very rarely, an adrenal carcinoma. It is characterized by hypertension
secondary to sodium retention, hypokalemia, and a decreased serum renin due to a negative feedback of
increased blood pressure on renin secretion.

Cushing's syndrome (choice A) is the result of increased glucocorticoid production, particularly cortisol. Physical
signs typically include "moon facies," truncal obesity, "buffalo hump," and purple abdominal striae.

Diabetes mellitus (choice B) is a condition of inadequate insulin production that presents with hyperglycemia
and ketoacidosis.

Pheochromocytoma (choice C) is a rare tumor of chromaffin cells occurring most commonly in the adrenal
medulla. The tumor secretes epinephrine and norepinephrine, resulting in secondary hypertension.

Secondary aldosteronism (choice E) results from an activation of the renin-angiotensin system caused by renal
ischemia, edema, and renal tumors. In contrast to primary aldosteronism, secondary aldosteronism is
associated with increased serum renin.




A 65-year-old man with hyperlucent lung fields develops extreme shortness of breath over a period of about 15
minutes. Chest x-ray shows a shift of the mediastinum to the right, and the lung field on the left appears even
more hyperlucent than before, with the exception of a white shadow near the heart border. Which of the following
is the most probable cause of the patient's current problem?


A. Bronchogenic carcinoma

B. Pleural effusion

C. Pulmonary embolism

D. Rupture of an emphysematous bulla

E. Tuberculosis


Explanation:

The correct answer is D. The patient's initial hyperlucent lung fields strongly suggest the presence of
emphysema. The radiologic findings after the increase in shortness of breath are consistent with free air in the
chest, which has collapsed the left lung and caused a shift in the location of the mediastinum. Such air might
have been introduced by rupture of a bulla. Small pneumothoraces are usually well tolerated, but larger ones
may require decompression (the needle from a syringe is sometimes used), or even surgical correction if bullae
continue to leak air.

Bronchogenic carcinoma (choice A) would be expected to produce a mass lesion.

Pleural effusion (choice B) usually develops slowly, and causes a whitening of lung fields when fluid is present.

Pulmonary embolism (choice C) can cause a sudden shortness of breath, but would not cause an increase in
the lucency of the lung fields.

Tuberculosis (choice E) would be expected to produce a mass lesion in the lung.



A 2-year-old infant with uncomplicated coarctation of the aorta appears to be in good health. Growth and
development are normal. The constriction is located just distal to the subclavian arteries. Which of the following
is decreased in this patient?


A. Blood flow in the upper body

B. Blood flow in the lower body

C. Blood pressure in the upper body

D. Vascular resistance in the upper body

E. Vascular resistance in the lower body


Explanation:

The correct answer is E. In fully compensated aortic coarctation, blood flow is normal in the upper and lower
body (choices A and B) despite an increased arterial pressure (about 50% higher) in the upper body (choice
C) compared to the pressure in the lower body. Because resistance = pressure/blood flow, it is clear that
resistance must be lower in the lower portions of the body. The mechanism of this decrease in resistance below
the constriction (and increased resistance above the constriction) is autoregulation of blood flow. The small
arteries and arterioles dilate (or constrict) in accordance with the metabolic needs of the tissues ensuring that
each tissue receive an adequate amount of blood flow. Thus, the increase in blood pressure in the upper body
leads to constriction of the arterioles, which increases vascular resistance (choice D), and the lower pressure
below the coarctation leads to dilation of the arterioles, which decreases vascular resistance in the lower body.


In which of the following conditions would oxygen therapy be most effective in alleviating hypoxia?


A. Anemia due to blood loss

B. Edematous tissues

C. Emphysema

D. Localized circulatory deficiencies

E. Right-to-left cardiac shunts


Explanation:

The correct answer is C. Chronic pulmonary emphysema is characterized by distention of small air spaces
distal to the respiratory bronchioles and destruction of alveolar septa. Long-term cigarette smoking is the usual
cause. The marked loss of lung parenchyma associated with emphysema leads to a decrease in the diffusion
capacity of the lungs, which reduces their ability to oxygenate blood and remove carbon dioxide. When arterial
hypoxemia is persistent and severe, oxygen therapy should therefore be considered.

Oxygen therapy is less effective for treating the hypoxia associated with anemia (choice A), edematous tissues
(choice B), localized circulatory deficiencies (choice D), and right-to-left cardiac shunts (choice E), because in
each case, there is already adequate oxygen available in the alveoli. The problem in each of these situations is
inadequate transport of oxygen to the tissues, blunting the effects of increasing the oxygen tension of the
inspired air. Oxygen therapy will nonetheless increase the amount of dissolved oxygen carried in the blood,
which may be life-saving in some instances.



A middle-aged woman comes to her physician's office with complaints of visual difficulties. A review of systems
and physical examination are unremarkable except for her eye exam. When a light is shined in her right eye,
there is no pupillary response in either eye. However, upon shining a light in her left eye, both ipsilateral and
contralateral pupillary responses are apparent. Her extraocular movements are intact. What is the most likely
location of her lesion?


A. Oculomotor nerve, left side

B. Oculomotor nerve, right side

C. Optic nerve, left side

D. Optic nerve, right side

E. Trochlear nerve, left side

F. Trochlear nerve, right side


Explanation:

The correct answer is D. Know your cranial nerves! This woman has a "Marcus-Gunn pupil," with a defect in the
afferent pathway of the optic nerve (in this case on the right side). Recall that the afferent limb of the pupillary
light reflex is the optic nerve (CN II); the efferent limb is the oculomotor nerve (CN III; parasympathetic fibers).
When light is shined into her right eye, because her right optic nerve is not functioning properly, the light signal
is not transmitted to the CNS, resulting in no pupillary response. As light is shined into her left eye, the left optic
nerve transmits the signal to the CNS, which then sends an outbound signal through both the right and left
oculomotor nerves to cause pupillary constriction in both eyes.

The oculomotor nerve (choices A and B) innervates all extraocular muscles except the lateral rectus
(innervated by the abducens nerve), and the superior oblique (innervated by the trochlear nerve; choices E
and F). The oculomotor nerve also mediates pupillary constriction (parasympathetic fibers), eyelid opening
(levator palpebrae), and innervates the ciliary muscle (allowing accommodation).

A patient with a lesion of the left optic nerve (choice C) would have no pupillary responses in either eye when
shining a light in the left eye; pupillary responses would be present in both eyes when shining a light in the right
eye.



On a routine physical examination for medical insurance, a midsystolic ejection murmur is detected in the
pulmonic area of a 35-year-old female executive. The cardiac examination also revealed a prominent right
ventricular cardiac impulse and wide and fixed splitting of the second heart sound. An EKG showed a right axis
deviation and chest x-ray showed enlargement of the right ventricle and atrium. Which of the following is the most
likely diagnosis?


A. Aortic stenosis

B. Atrial septal defect

C. Mitral regurgitation

D. Mitral stenosis

E. Pulmonary valve stenosis


Explanation:

The correct choice is B. The classic findings in atrial septal defect are a prominent right ventricular cardiac
impulse, a systolic ejection murmur heard in the pulmonic area and along the left sternal border, and fixed
splitting of the second heart sound. These findings are due to an abnormal left to right shunt through the defect,
creating a volume overload on the right side. The increase in volume on the right side creates the flow murmur,
the dilatation of the right-sided chambers, and the delayed closure of the pulmonic valve, all of which are
present in this case.

Aortic stenosis (choice A) is also associated with a systolic ejection murmur. The murmur is usually loudest at
the right sternal border and radiates upward to the jugular notch. This condition is associated with left
ventricular hypertrophy.

Mitral regurgitation (choice C) would present with a systolic murmur as well. However, left atrial enlargement
would be seen before right ventricular enlargement.

Mitral stenosis (choice D) would present with an "opening snap" and a diastolic murmur.

Pulmonary valve stenosis (choice E) causes an increase in right ventricular pressure resulting in right
ventricular hypertrophy and pulmonary artery dilatation. A crescendo-decrescendo murmur may be heard if the
stenosis is severe. Right atrial enlargement would not be present.


During a boxing match, a contestant is "knocked out" by a blow to the lateral skull. He recovers after a few
minutes, and is asymptomatic for the next 12 hours. He then develops a severe headache, changes in mental
status, nausea, and vomiting. Which of the following is the most likely diagnosis?


A. Basilar skull fracture

B. Epidural hemorrhage

C. Intracerebral hemorrhage

D. Subarachnoid hemorrhage

E. Subdural hematoma


Explanation:

The correct answer is B. All of the lesions listed in the answers can occur in brain trauma. However, the
scenario described is classic for epidural hemorrhage. In this scenario, a severe blow to the lateral skull causes
both skull fracture and laceration of the middle meningeal artery (know the name of this artery, as it is
frequently tested), leading to a momentary loss of consciousness, which is followed by a lucid (asymptomatic)
period of 1-48 hours before the patient's neurologic condition deteriorates. This scenario is frequently tested in
examinations, but you should be aware that in real clinical life, there may be no initial loss of consciousness and
the patient may not have sought medical help for skull trauma.

Basilar skull fractures (choice A) are usually located in the vicinity of the petrous bone or along the sphenoid
bone. Clinical signs of basilar skull fracture include hemotympanum (blood visible behind the tympanic
membrane), delayed ecchymosis over the mastoid process (Battle's sign), or periorbital ecchymosis ("raccoon
sign"). CSF leakage (sometimes from the nose), or pneumocephalus may also occur.

Intracerebral hemorrhage (choice C) is most often due to hypertension, anticoagulant use, cerebral amyloid
angiopathy, or cocaine and/or methamphetamine abuse.

Subarachnoid hemorrhage (choice D) is most commonly caused by rupture of a cerebral berry aneurysm, but
may also be associated with arteriovenous malformations or intraparenchymal hemorrhage.

Subdural hematomas (choice E) are due to rupture of the bridging veins between the periosteal dura and the
superficial cerebral veins. Blood accumulates below the dura, producing symptoms of increased intracranial
pressure in minutes to hours (acute subdural hematomas), or after weeks or months (chronic subdurals, often
seen in the elderly). An acute subdural may present like an epidural hemorrhage, but a forceful blow to the
lateral skull producing skull fracture is generally associated with an epidural, rather than a subdural, bleed.





A 77-year-old black male is recovering in a rehabilitation center 10 days after having sustained a massive
anterolateral infarction of the left ventricle. He suddenly develops paralysis of the entire right half of the body,
including facial and eye muscles. Which of the following complications of myocardial infarction is most likely to
have precipitated this event?


A. Calcific coronary atherosclerosis

B. Electromechanical dissociation

C. Left bundle branch block

D. Myocardial rupture

E. Ventricular mural thrombus


Explanation:

The correct answer is E. This patient has suffered a cerebrovascular accident as a consequence of
thromboemboli emanating from a mural thrombus formed over the recent myocardial infarction. Mural thrombus
frequently develops over a previously infarcted segment of myocardium, especially when the infarction is large
and a ventricular aneurysm develops. Ischemic damage to the endocardium, soluble factors released by the
injured myocardium, and altered wall kinetics that produce sluggish blood flow all favor mural thrombus
formation. One important consequence of a mural thrombus is thromboembolism to the systemic circulation,
producing a stroke, as in this patient's case.

Calcific coronary atherosclerosis (choice A) is very likely responsible for the patient's original myocardial
infarction. Coronary atherosclerosis is not a consequence of myocardial infarction, and it does not directly
predispose to the development of embolic strokes. Complicated atherosclerotic lesions in the carotid or cerebral
circulation, however, may directly lead to a cerebrovascular accident.

Electromechanical dissociation (EMD; choice B) is a catastrophic event that frequently leads to sudden death,
not stroke. In EMD, although a normal potential is transmitted through the myocardium, no pulse of arterial
blood is sent to the systemic circulation. EMD may occur as a consequence of pericardial tamponade, massive
pulmonary embolism, or myocardial toxins that prevent normal cardiac muscle contraction.

Left bundle branch block (choice C) represents failure of electrical transmission along the Purkinje fibers to the
left ventricular myocardium. Bundle branch blocks are important causes of arrhythmia, but are not responsible
for cerebrovascular accidents.

Myocardial rupture (choice D) is an infrequent consequence of myocardial infarction that typically occurs
approximately 1 week after the infarction. If scarring of the infarcted segment does not keep pace with necrosis
and digestion of the infarcted tissue, the myocardium can rupture under the high intraventricular pressures, and
pericardial tamponade generally ensues. Myocardial rupture rapidly leads to acute heart failure, not stroke.



Pulmonary artery pressure:
22/7 mm Hg
Pulmonary wedge pressure:
10 mm Hg
Left ventricular pressure:
170/5 mm Hg
Aortic pressure:
120/80 mm Hg


A 50-year-old male complaining of chest pain is admitted to the emergency room. He is taken to the cardiac
catheterization laboratory on the same day. Selected pressures are shown above. What is the most likely
diagnosis?


A. Aortic regurgitation

B. Aortic stenosis

C. Mitral regurgitation

D. Mitral stenosis


Explanation:

The correct answer is B. This patient has aortic stenosis. In aortic stenosis, the blood is ejected from the left
ventricle through a smaller-than-normal opening. Because the resistance to ejection of blood is high, the left
ventricular pressure can sometimes increase to over 250 mm Hg with normal systolic pressure in the aorta.
Note that in this patient, the left ventricular peak systolic pressure has increased to 170 mm Hg, the aortic
systolic pressure is normal at 120 mm Hg, and a 50 mm Hg gradient exists across the aortic valve during
systole.

In aortic regurgitation (choice A), blood flows backward through the aortic valve during diastole when the valve
is closed. Left ventricular systolic pressure and aortic systolic pressure are nearly the same with pure aortic
regurgitation.

Mitral regurgitation (choice C) means there is backward flow of blood through the mitral valve during systole.
This accumulation of extra blood in the left atrium during ventricular systole leads to an elevation in the
pulmonary wedge pressure (which is used as an estimate of left atrial pressure).

In mitral stenosis (choice D), blood must flow from the left atrium into the left ventricle through a
smaller-than-normal opening. Because the resistance to blood flow through the mitral valve is higher than
normal, the left atrial pressure (estimated by the pulmonary wedge pressure) is often several mm Hg higher
than the left ventricular end diastolic pressure.



A 63-year-old man with essential hypertension has gone several weeks without taking his medications. He arrives
at the emergency room with a severe laceration on his right hand after falling on a broken beer bottle. He has a
heart rate of 90 beats per minute and a blood pressure of 170/115. Which of the following is most likely to be
decreased in the skeletal muscles of his legs?


A. Adenosine levels

B. Arterial blood pressure

C. Arteriolar resistance

D. Blood flow

E. Venous oxygen concentration


Explanation:

The correct answer is A. The skeletal muscles of the body have a normal blood flow even when blood pressure
is chronically elevated. Organs and tissues in which the vasculature has primarily a nutritive function (e.g.,
brain, heart, and skeletal muscle) regulate their blood flow in accordance with the metabolic needs of the
tissues. These tissues exhibit short-term autoregulation of blood flow such that the increase in flow caused by
an elevated arterial pressure is minimized by constriction of the arterioles. The constriction is caused in part by
decreased levels of adenosine (an endogenous vasodilator) in the tissues. The rate of adenosine production in
a tissue is a function of its metabolic rate, which is not affected significantly by an increase in systemic
pressure. When blood flow to the muscle increases, the adenosine is literally washed from the muscle, lowering
the tissue levels of adenosine. The decrease in adenosine concentration causes small arteries and arterioles in
the muscle to constrict and this increase in resistance (choice C) maintains blood flow (choice D) at a normal
rate in the face of increased arterial pressure (choice B). The overall process is called autoregulation of blood
flow.

Venous oxygen concentration (choice E) does not decrease in the skeletal muscles of hypertensives because
blood flow is maintained at an adequate level to meet the nutritional demands of the muscles.



Which of the following conditions would mostly likely be associated with chronic gastritis (Type A) resulting from
autoimmune destruction of parietal cells?


A. Decreased growth of luminal bacteria

B. Decreased likelihood of developing gastric carcinoma

C. Decreased plasma concentration of gastrin

D. Increased production of macrocytic red blood cells

E. Increased secretion of pancreatic bicarbonate


Explanation:

The correct answer is D. Autoimmune destruction of parietal cells would lead to decreased secretion of gastric
acid and intrinsic factor. The diminished availability of intrinsic factor would result in poor absorption of dietary
vitamin B12. Over time, the vitamin B12 deficiency could lead to pernicious anemia, which is characterized by
increased production of macrocytes (megaloblasts) by the bone marrow.

Because of the decrease in gastric acid secretion, luminal bacteria (choice A) would most likely exhibit
increased (not decreased) growth. One of the functions of HCl secreted by the parietal cells is to sterilize the
gastric lumen.

Patients with Type A gastritis have an increased likelihood of developing gastric carcinoma (not decreased,
choice B).

A decrease in acid secretion leads to increased secretion of gastrin (not decreased, choice C) by antral G
cells. This is because low gastric pH (less than 3) inhibits gastrin secretion via paracrine release of
somatostatin from cells in the gastric mucosa that can sense the acidity. With decreased parietal cells, the pH
of the gastric lumen would rise and remove this inhibitory component.

Because less acid would be delivered to the duodenum with parietal cell destruction, less secretin would be
released into the blood.. This would result in decreased pancreatic bicarbonate secretion (not increased,
choice E).



A 55-year-old hypertensive man develops sudden onset of excruciating pain beginning in the anterior chest, and
then radiating to the back. Over the next 2 hours, the pain moves downward toward the abdomen. Which of the
following is the most probable diagnosis?


A. Aortic dissection

B. Aortic valve stenosis

C. Atherosclerotic aneurysm

D. Myocardial infarction

E. Syphilitic aneurysm


Explanation:

The correct answer is A. This patient has an aortic dissection (formerly called dissecting aneurysm), a
potentially fatal condition that is too often confused clinically with myocardial infarction. The most important
clinical clue is that the pain shifts with time. Non-invasive techniques such as transesophageal
echocardiography, computed tomography (CT), and magnetic resonance imaging (MRI) are increasingly useful
in making this diagnosis.

Aortic valve stenosis (choice B) would not be expected to produce severe chest pain of acute onset.

This patient's clinical history does not suggest either an atherosclerotic (choice C) or a syphilitic (choice E)
aneurysm. Even if he had one of either of these types of aneurysms and it had begun to rupture, the distinctive
feature of severe pain moving downward would probably not be present.

Myocardial infarction (choice D) is the major diagnosis most often confused with this patient's condition. The
movement of the pain is the major clinical tip-off suggesting that this is not the correct answer.



A neurologist gives a car accident victim a neurological examination. As part of the examination, he presents the
patient with a picture of a dog and asks her to talk about it. She seems unable to name or recognize the picture.
He then asks her to copy a picture of the dog, which she is able to do, although she still is unable to identify the
animal as a dog. Which of the following is the most appropriate diagnosis?


A. Agnosia

B. Alexia

C. Aphasia

D. Apraxia

E. Dyslexia


Explanation:

The correct answer is A. Agnosia is the inability to recognize, despite adequate sensation. This patient has a
type of agnosia known as visual agnosia, which is the inability to recognize familiar objects despite the ability to
see. The fact that she could copy a picture of the dog indicates that she has adequate visual acuity, visual
fields, and perception. If this patient had only visual agnosia, if she were to hear a dog bark (auditory) or pet a
dog (tactile), then she would be able to recognize and name the dog. Prosopagnosia is a special type of visual
agnosia in which a person is unable to recognize faces. There are also other types of agnosias, including
auditory and tactile agnosias.

Alexia (choice B) is an acquired reading disorder due to brain damage. People with this disorder can
comprehend words that are spelled out loud, and can understand words and letters that are written on the
palm, but they cannot read.

Aphasia (choice C) is an acquired disorder of language due to brain damage. Examples are Broca's aphasia
and Wernicke's aphasia.

Apraxia (choice D) is the loss of the ability to carry out certain movements correctly in response to stimuli that
normally elicit them. This occurs even though the patient has no sensory loss, no weakness, and no
disturbance of language comprehension.

Dyslexia (choice E) is a developmental reading disorder.


A 62-year-old man is admitted to the hospital because of symptoms of congestive heart failure, which have been
present for the past few years. Fifteen years before admission, the man sustained a knife wound to the left
supraclavicular area. Physical examination reveals signs of a large, left subclavian arteriovenous fistula. Which
of the following is expected to be present in this man?


A. Decreased heart rate

B. Decreased stroke volume

C. Decreased mixed venous oxygen content

D. Increased resting cardiac output

E. Increased diastolic blood pressure

F. Increased systemic vascular resistance


Explanation:

The correct answer is D. Increased cardiac output can be demonstrated when there is a large fistula that
involves a major artery such as the aorta, subclavian artery, femoral artery, common carotid artery, or iliac
artery. The increase in cardiac output caused by the fistula is roughly equal to the blood flow through the
fistula.

The increase in cardiac output is associated with increases in both heart rate (choice A) and stroke volume
(choice B).

The mixed venous oxygen content (choice C) is increased because oxygenated arterial blood flows through the
fistula into the venous system.

The diastolic blood pressure (choice E) falls because blood can rapidly exit the arterial system through the
fistula, but mean blood pressure is maintained relatively constant secondary to increased blood volume caused
by renal retention of salt and water.

Because blood is shunted from an artery to a vein through a low resistance pathway, the systemic vascular
resistance (choice F) is decreased.




A couple presents to a clinic for work-up of infertility after 5 years of unprotected intercourse. The wife denies
any medical problems and notes regular menstrual cycles. The husband states that he has had chronic sinusitis
and lower respiratory tract infections. Physical examination of the woman is unremarkable. Examination of the
man is remarkable for dextrocardia. Further work-up of the husband will most likely reveal


A. azoospermia

B. germinal cell aplasia

C. immotile sperm

D. isolated gonadotropin deficiency

E. varicocele


Explanation:

The correct answer is C. The husband is suffering from Kartagener's syndrome, an autosomal recessive
disorder characterized by infertility, situs inversus, chronic sinusitis, and bronchiectasis. The underlying cause
of these varied manifestations are defects in the dynein arms, spokes of microtubule doublets of cilia in the
airways and the reproductive tract. Since sperm motility is dependent on the functioning of cilia, infertility
frequently accompanies this disorder. Situs inversus occurs because ciliary function is necessary for cell
migration during embryonic development.

Azoospermia (choice A) is not a feature of Kartagener's syndrome, as sperm production or survival is not
affected in this disorder.

Germinal cell aplasia (choice B), also known as Sertoli only syndrome, is characterized by oligospermia or
azoospermia.

Isolated gonadotropin deficiency (choice D) is characterized by delayed or incomplete pubertal maturation.

Varicocele (choice E) results in an increased testicular temperature, decreasing the count of normal, viable
sperm.



A patient with long-standing constipation enters a clinical research study. After a complete physical examination,
a small intraluminal balloon is inserted through the anus to the rectum. Transducers are also inserted to measure
internal and external anal sphincter pressures. Inflation of the rectal balloon causes the external anal sphincter to
contract, but the internal anal sphincter, which exhibits normal tone, fails to relax and the urge to defecate is not
sensed. Which of the following structures is most likely damaged?


A. Internal anal sphincter

B. External anal sphincter

C. Pelvic nerve

D. Pudendal nerve

E. Rectum


Explanation:

The correct answer is C. The defecation reflex that is evoked when the rectum is distended involves three
responses: 1) the internal anal sphincter relaxes, 2) the external anal sphincter contracts, and 3) a conscious
urge to defecate is perceived. This neural reflex involves the pelvic nerve, which provides the parasympathetic
preganglionic innervation to the internal anal sphincter (composed of smooth muscle) and also carries the
sensory afferent information from the rectum to the spinal cord, and the pudendal nerve, which carries the
somatic efferent inputs to the external anal sphincter (composed of skeletal muscle). In the patient described
above, the pelvic nerve is most likely damaged since neither the reflex relaxation of the internal anal sphincter
nor the urge to defecate is evoked by rectal distention.

Damage to the internal anal sphincter (choice A) would most likely cause resting tone to be low and, if anything,
lead to fecal incontinence rather than constipation. Furthermore, damage to the internal anal sphincter could
not explain the failure of the appearance of the urge to defecate.

The external anal sphincter (choice B) appears to function normally since distention of the rectum evokes the
expected contraction. The normal contraction of the external anal sphincter also suggests that the pudendal
nerve (choice D) is intact.

Since distention of the rectum (choice E) evoked the contraction of the external anal sphincter, it too appears to
be functioning normally.



A 60-year-old man complains of difficulty arising from a chair and initiating new movements. On exam, you notice
a resting hand tremor and cogwheel rigidity. Which of the following amino acids is the precursor for the
neurotransmitter that is deficient in the brain of this patient?


A. Glutamate

B. Glycine

C. Histidine

D. Tryptophan

E. Tyrosine


Explanation:

The correct answer is choice E. This question requires three steps of logic. First, figure out the diagnosis (a
classic case of Parkinson's disease), then remember which neurotransmitter is involved in the disease
(dopamine), and finally, recall which amino acid serves as the precursor for that neurotransmitter (tyrosine).
The hydroxylation of tyrosine by tyrosine hydroxylase results in DOPA, which is then decarboxylated to
dopamine. Note that norepinephrine and epinephrine are also tyrosine derivatives, as are the melanins and the
thyroid hormones, thyroxine and triiodothyronine.

The other answer choices are also precursors of specialized products:

Glutamate (choice A) can be converted to the inhibitory neurotransmitter GABA by the action of glutamate
decarboxylase.

Glycine (choice B) is involved in the synthesis of both creatine (along with arginine and S-adenosylmethionine)
and heme (along with succinyl-CoA).

Histidine (choice C) can be decarboxylated to histamine, an important inflammatory mediator.

Tryptophan (choice D) can be converted to serotonin by a hydroxylation reaction (tryptophan hydroxylase)
followed by a decarboxylation reaction.


A 40-year-old woman presents to the emergency department because of hematuria. Laboratory analyses show
significant proteinuria, bacteria and white cells in the urine, and a blood urea nitrogen (BUN) of 40 mg/dL with a
creatinine of 4.0 mg/dL. Ultrasonography reveals enlarged kidneys, and she is given a provisional diagnosis of
polycystic renal disease. Which of the following is likely to be decreased in this patient?


A. Creatinine clearance

B. Extracellular sodium concentration

C. Glucose clearance

D. Plasma creatinine levels

E. Plasma inulin levels


Explanation:

The correct answer is A. Polycystic kidney disease leads to progressive decrements in renal function,
eventually resulting in renal failure (evidenced by the increased BUN and creatinine). These abnormalities are
caused by a reduction in glomerular filtration rate (GFR), which produces a decrease in creatinine clearance.
The production of creatinine, a waste product of metabolism, bears a direct relation to the muscle mass of an
individual and is independent of renal function. Because creatinine is freely filtered by the glomerulus, but not
secreted or reabsorbed to a significant extent, the renal clearance of creatinine is approximately equal to the
GFR. Therefore, creatinine clearance is commonly used to assess renal function in the clinical setting.

The extracellular sodium concentration (choice B) is not expected to change significantly in this patient.

Glucose (choice C) is not normally excreted, so glucose clearance is normally zero. Thus, is not possible for
glucose clearance to be decreased.

Plasma creatinine concentration (choice D) increases when GFR decreases.

Inulin clearance is used to estimate GFR. However, inulin is foreign to the body, and inulin plasma levels
(choice E) are normally zero.



Two diabetic patients are seen by a clinician. The first patient is a 16-year-old boy who 2 years previously had
presented with polyuria and polydipsia. The second patient is a 65-year-old woman whose diabetes was
identified by the presence of hyperglycemia on a routine blood glucose screen 10 years previously. Compared to
the 65-year-old patient with diabetes, the 16-year-old diabetic is more likely to


A. be obese

B. become euglycemic with oral hypoglycemic agents

C. develop ketoacidosis

D. have relatively high endogenous insulin levels

E. not have the HLA-DR3 or HLA-DR4 allele


Explanation:

The correct answer is C. The 16-year-old probably has type 1 (juvenile onset) diabetes mellitus, while the
65-year-old probably has type 2 (maturity onset) diabetes mellitus. These two types of diabetes differ in many
respects. Ketoacidosis is more apt to develop in type 1 diabetes.

type 2 diabetics tend to be obese (choice A), while type 1 diabetics are often thin.

type 1 is usually apparently due to viral or immune destruction of beta cells, while type 2 is apparently usually
due to increased resistance to insulin; consequently the 65-year-old, rather than the 16-year-old, is more likely
to have relatively high endogenous levels of insulin (choice B).

type 2 diabetes can often be controlled with oral hypoglycemic agents (choice C), while type 1 diabetics
generally require insulin. Note that some type 2 diabetics also may require insulin as the disease evolves.

type 1 diabetes has a strong association with HLA-DR3 and HLA-DR4 (choice E), while type 2 does not have
any strong HLA associations.




A 69-year-old man involved in a motorcycle accident loses an estimated 1 L of blood from a severed artery in his
leg. Constriction of which of the following types of blood vessels is most important for minimizing the decrease in
mean systemic filling pressure caused by this blood loss?


A. Aorta and large arteries

B. Arterioles

C. Capillaries

D. Small arteries

E. Venules and veins


Explanation:

The correct answer is E. The venous system serves as an important blood reservoir for the circulation. When
blood is lost from the body and blood pressure begins to fall, pressure reflexes are elicited that send
sympathetic nerve signals to venules and veins causing them to constrict. By "tightening" the circulation and
taking up much of the "slack" caused by the blood loss, nearly normal function can usually be restored with up
to a 20% loss of blood volume.

The mean systemic filling pressure (MSFP) is the pressure that exists in all parts of the circulation when the
heart has been stopped and the blood volume has become redistributed in the system until all pressures are at
equilibrium. The MSFP is thus a measure of the "tightness" with which the circulatory system is filled with blood.
The more the system is filled (i.e., when MSFP increases), the easier it is for blood to flow into the heart, which
tends to increase venous return.

Constriction of the arterial system has relatively little effect on mean systemic filling pressure because the
arterial system (choices A, B, and D) contains a relatively small volume of blood.

Capillaries do not constrict because they do not contain smooth muscle cells in their walls (choice C).



A 20-year-old male is evaluated for persistent gynecomastia. Physical examination reveals the presence of
abnormally small, firm testes. Testicular biopsy shows fibrosis and hyalinization of the seminiferous tubules. The
Leydig cells are present in clumps and are hyperplastic. Chromosomal analysis shows the presence of an XXY
genotype. Which of the following laboratory findings would be most likely in this individual?


A. Decreased plasma estrogen

B. Decreased plasma follicle stimulating hormone

C. Decreased plasma luteinizing hormone

D. Decreased plasma testosterone

E. Increased plasma inhibin


Explanation:

The correct answer is D. Klinefelter's syndrome is characterized by an XXY genotype and is associated with
seminiferous tubule dysgenesis. In this disorder, there are usually few symptoms before puberty, which may be
delayed. At puberty, the seminiferous tubules fail to enlarge normally and instead undergo fibrosis and
hyalinization. The result is an inability to produce sperm. The Leydig cells are hyperplastic and clumped
together, and do not function normally (secretion of testosterone is decreased). As a consequence, plasma
levels of LH are increased (not decreased, choice C) due to the loss of feedback inhibition. The increased LH
stimulates the Leydig cells to increase estrogen (not decreased, choice A) secretion. The increased
estrogen:testosterone ratio is responsible for the gynecomastia, small penis, sparse body hair, and other
feminized features of Klinefelter's syndrome. Because of dysgenesis of the seminiferous tubules, the secretion
of inhibin is reduced (not increased, choice E). Low plasma levels of inhibin result in increased plasma FSH (not
decreased, choice B) due to loss of feedback inhibition at the pituitary.



[IMG]file:///C:/DOCUME%7E1/Owner/LOCALS%7E1/Temp/msohtml1/01/clip_image003.jpg[/IMG]
.

The pressure tracings from the thoracic and abdominal aorta shown above were obtained from a 3-month-old infant
who exhibited dyspnea, difficulty feeding, and poor weight gain. Physical examination reveals a weak femoral pulse
compared to the radial pulse. Which of the following is likely to be higher than normal in this infant?


A. Femoral artery wall thickness

B. Abdominal aorta wall thickness

C. Left ventricular wall thickness

D. Resting blood flow in leg muscles

E. Renal blood flow


Explanation:

The correct answer is C. The infant has a coarctation of the aorta. In this condition, the heart must pump against a
higher-than-normal pressure. The left ventricle responds to this increased pressure load by undergoing hypertrophy,
i.e., the wall thickness of the left ventricle increases as the myocytes enlarge. A similar process occurs when there is
systemic hypertension. The increased blood pressure in the upper body (above the coarctation) also produces
arterial hypertrophy, increasing the thickness of the vessel walls.

The wall thickness of the abdominal aorta (choice B), femoral artery (choice A), and other arteries below the
constriction may decrease in response to the lower than normal pressures.

Although one might predict a lower-than-normal blood flow to the kidneys (choice E), leg muscles (choice D), and
other organs below the coarctation, if the body can compensate fully for the coarctation, blood flow will be normal in
these low-pressure areas of the body.


A 45-year-old man suffering from glomerulonephritis has a creatinine clearance of 50 mL/min. His medical
records indicate that his creatinine clearance was 100 mL/min about 1 year ago. Assuming that there has been
no change in his diet, which of the following changes can be expected in this patient compared to 1 year ago?


A. A 2-fold decrease in blood urea nitrogen concentration

B. A 2-fold decrease in creatinine excretion rate

C. A 2-fold increase in creatinine excretion rate

D. A 2-fold increase in creatinine reabsorption

E. A 2-fold increase in plasma creatinine concentration


Explanation:

The correct answer is E. Creatinine clearance is used clinically to estimate glomerular filtration rate (GFR).
Therefore, the 50% decrease in creatinine clearance in this patient suggests that GFR has decreased by 50%
over the past year. Because creatinine is freely filtered but not reabsorbed (choice D), the filtration rate and
excretion rate of creatinine are equal during steady state conditions. When GFR decreases, the rate of
creatinine excretion will also decrease, causing the rate of creatinine excretion to fall below the rate of
creatinine production. The result is an increase in plasma creatinine concentration. When plasma creatinine
levels have increased by 2-fold, normal amounts of creatinine can then again be excreted (compare to choices
B and C) because the excretion rate of creatinine is equal to the product of GFR and plasma creatinine
concentration. In summary, when GFR decreases, the plasma creatinine concentration continues to increase
until the rate of creatinine filtration (and excretion) becomes equal to the rate of creatinine production by the
body.

The blood urea nitrogen concentration (choice A) increases when GFR is reduced.



A 42-year-old Caucasian woman with no history of prior surgery complains of extreme weakness and fatigability
that has persisted for the past 5 months. She reports nausea and stomach cramps and has had trouble keeping
solid foods down. She is hypotensive while sitting, and her blood pressure falls even more upon standing. She
notes increased freckling around her eyes, and on examination, her palmar creases appear darkened. Serum
potassium is 6.5 mEq/L. Which of the following is the most likely diagnosis?


A. Addison's disease

B. Conn's syndrome

C. Cushing's syndrome

D. Nelson's syndrome

E. Secondary adrenal insufficiency

F. Tertiary adrenal insufficiency


Explanation:

The correct answer is A. Addison's disease usually occurs because of autoimmune destruction of the adrenal
cortex (all three zones are typically involved), resulting in decreased secretion of cortisol, aldosterone, and
adrenal androgens. Hyperpigmentation is the classical physical finding, resulting from increased serum ACTH
due to loss of negative feedback inhibition by cortisol at the pituitary and/or hypothalamus. The increase in
pigmentation may occur because the first 13 amino acids of ACTH are identical to alpha-melanocyte stimulating
hormone. Low serum levels of cortisol produce gastrointestinal symptoms such as nausea, vomiting, and
anorexia. Fatigability and weakness are almost always reported. Blood pressure is usually low and orthostatic
hypotension may be present, because arterioles are less responsive to the constrictor effects of
catecholamines in the absence of cortisol. The cardiovascular symptoms are worsened by the loss of blood
volume due to aldosterone deficiency. Hyperkalemia is a manifestation of the low serum aldosterone;
hyponatremia may also be present.

Conn's syndrome (choice B) results from hypersecretion of aldosterone. It is characterized by hypertension,
hypernatremia, and hypokalemia.

Cushing's syndrome (choice C) occurs because of excessive secretion of cortisol. It is characterized by central
obesity, buffalo hump, moon facies, hypertension, and if anything, hypokalemia.

Nelson's syndrome (choice D) results in extreme hyperpigmentation. It can occur in patients who have had
adrenalectomy to treat Cushing's disease. It results because of excessive secretion of ACTH from a pituitary
adenoma that is no longer being restrained by the suppressive effect of cortisol. The patient mentioned above
has no history of adrenalectomy.

Both secondary (choice E) and tertiary (choice F) adrenal insufficiency result in low serum levels of ACTH. The
subsequent hypocortisolism can produce the gastrointestinal complaints and fatigability, but not hyperkalemia.
With deficiency of CRH or ACTH, serum aldosterone usually remains in the normal range, and signs of
mineralocorticoid deficiency are not present. Furthermore, low serum levels of ACTH would not produce
hyperpigmentation.



A 64-year-old woman had a cerebrovascular accident 6 months ago. Her past medical history is remarkable for
hyperthyroidism and atrial fibrillation. She initially presented with slurred speech and right hemiparesis. The
hemiparesis resolved, but her speech is still agrammatic and nonfluent, and she has difficulty finding words and
completing sentences. Her comprehension is intact, and she appears frustrated when she attempts to speak.
The remainder of the neurologic examination is normal. Which of the following best describes her deficit?


A. Apraxia

B. Broca aphasia

C. Dysarthria

D. Global aphasia

E. Wernicke aphasia


Explanation:

The correct answer is B. Broca aphasia, caused in this case by an embolus to the Broca area (inferior frontal
gyrus), is often associated with hemiparesis. The aphasia is characterized by slow, nonfluent speech with
deficits in word finding. Because comprehension is normal, patients are typically aware of the problem and
appear frustrated.

Apraxia (choice A) is a deficit of purposeful movement caused by a central lesion.

Dysarthria (choice C) is a deficit in speech articulation with normal grammar and word finding.

Global aphasia (choice D) is usually caused by large infarcts in the distribution of the middle cerebral artery. It
is characterized by elements of Broca and Wernicke aphasia (i.e., an inability to generate or comprehend
fluent speech).

Wernicke aphasia (choice E), caused by lesions in Wernicke area, is characterized by fluent speech that has a
normal tempo, but is filled with incorrect words and neologisms. The patient is unable to comprehend speech.


A 44-year-old woman presents with a chief complaint of a sharp, stabbing pain in her chest for the past 12 hours.
A careful history reveals that the patient experienced a myocardial infarct 3 years ago. The woman refuses to lie
down in the examining room and instead leans forward, stating that it allows her to breathe more easily. Physical
examination is unremarkable. The ECG demonstrates diffuse ST segment elevations with upright T waves. Chest
radiographs appear normal. Creatine kinase (MB fraction) is normal. Which of the following is the most likely
diagnosis?


A. Acute pericarditis

B. Dissecting aortic aneurysm

C. Myocardial infarction

D. Stable angina

E. Unstable angina


Explanation:

The correct answer is A. All of the answer choices represent common cardiovascular causes of chest pain.
However, only pericarditis and dissecting aortic aneurysms will produce sharp, knife-like pains. Patients with
pericarditis relieve their pain by sitting and leaning forward. The characteristic ECG patterns of pericarditis
include diffuse ST elevations with upright T waves. While a pericardial rub is diagnostic of pericarditis, its
presence is not necessary for diagnosis, and the physical examination may well be unrevealing. Typically, in
uncomplicated pericarditis, both chest radiographs and cardiac isoenzyme levels appear normal.

Pericarditis can be differentiated from dissecting aortic aneurysms (choice B) on the basis of clinical findings.
The pain associated with dissecting aortic aneurysms is usually unrelated to breathing, while the pain
associated with pericarditis is related to breathing.

Myocardial infarcts (choice C), as well as stable (choice D) and unstable angina (choice E), typically produce
more visceral types of pain.





A 30-year old male complains of fatigue and diffuse skeletal pain. He has a history of Crohn's disease, which led
to resection of the terminal ileum 3 years previously. Steatorrhea and diarrhea have continued since the surgery.
Laboratory tests show that serum calcium is 7.5 mg/dL, serum phosphate is 2.5 mg/dL, and serum parathyroid
hormone (N-terminal) is 750 pg/mL. Which of the following is the most likely cause of these findings?


A. Osteoporosis

B. Paget's disease

C. Primary hypoparathyroidism

D. Renal failure

E. Vitamin D deficiency


Explanation:

The correct answer is E. Whenever serum calcium and phosphate are both decreased, vitamin D deficiency
should be considered. In this case, the vitamin D deficiency is due to fat malabsorption, including the fat-soluble
vitamin D, subsequent to ileal resection. If more than 100 cm of the ileum are removed, primary bile acid
production by the liver cannot keep up with bile salt loss in the stool. The total bile salt pool decreases and fat
absorption, including the fat-soluble vitamins is poor. Serum calcium is low because of decreased dietary
absorption. Serum parathyroid hormone increases in response to the low calcium. Serum phosphate is low
because of decreased dietary absorption and increased renal excretion (due to the increased parathyroid
hormone). With vitamin D deficiency, the bones demineralize, producing osteomalacia. Clinical manifestations
often go unnoticed. Vague complaints of weakness and bone pain may be present. Radiographs of bones in
osteomalacia typically reveal the presence of pseudofractures along the inner aspects of the femur, the pubic
rami, and the outer edges of the scapulas, upper fibula, and metatarsals. These radiolucent bands, which are
perpendicular to the bone surface, may occur because of pulsations of major arteries that cross the bone.

Osteoporosis (choice A) is characterized by loss of bone mass, both matrix and mineral. It is usually
asymptomatic, and serum levels of calcium, phosphate, and parathyroid hormone are within the normal range.
The first hint of bone loss comes because of a fracture in the wrist, hip, or vertebra. Dual-energy radiography,
or other similar techniques, can be used to directly quantify the degree of bone loss.

In Paget's disease (choice B), bone mineral turnover is increased compared to normal. Its cause is unknown,
but may be due to a slow virus that infects osteoclast cells. Both osteoblast and osteoclast activity is increased
in focal areas of the bone. The disease is usually asymptomatic, the chief complaint being bone pain over the
lesions. Laboratory findings include increased serum alkaline phosphatase, but serum calcium and parathyroid
hormone levels are usually normal.

With primary hypoparathyroidism (choice C), serum calcium is decreased and serum parathyroid hormone is
decreased, but serum phosphate is also increased (not decreased). Serum phosphate is increased because
urinary excretion is diminished subsequent to the decrease in parathyroid hormone.

In renal failure (choice D), an increase (not decrease) in serum phosphate subsequent to decreased urinary
excretion is a primary manifestation. Serum calcium is decreased because the hyperphosphatemia drives the
equilibrium between calcium and phosphate toward hydroxyapatite crystals. This, in turn, produces an increase
in parathyroid hormone secretion with subsequent bone demineralization (renal osteodystrophy).



A three-year-old child is brought to the emergency room after inhaling a peanut. The peanut has lodged in the
right mainstem bronchus, largely occluding it. The child is cyanotic, and non-invasive transcutaneous monitoring
reveals a PO2 of 60 mm Hg. Which of the following mechanisms best accounts for the child's hypoxemia?


A. Decreased capacity of pulmonary diffusion

B. Decreased PO2 in inspired air

C. Hypoventilation of central origin

D. Hypoventilation of peripheral origin

E. Inequalities of ventilation and perfusion


Explanation:

The correct answer is E. Inequalities of ventilation and perfusion contribute to hypoxia in many settings. In this
case, blood goes to both lungs (perfusion), but air is prevented from entering one of the lungs (ventilation).
Because the right lung is being perfused, but not ventilated, hypoxemia ensues when the deoxygenated blood
from the right lung mixes with oxygenated blood from the left lung. If the inadequate ventilation of the lung
persists long enough, the lung tissue itself can be damaged, causing a secondary local dilation of arterioles,
making the problem even worse. Peanuts are notorious for producing this type of problem in young children
because of their size and shape, which allows them to lodge in the trachea or main bronchus after aspiration.

Decreased diffusion capacity (choice A) can occur when the blood-gas barrier is thickened (e.g., diffuse
interstitial fibrosis, sarcoidosis, asbestosis, respiratory distress syndrome), when the surface area of the
blood-gas barrier is reduced (e.g., pneumonectomy, emphysema), or when less hemoglobin is available to
carry oxygen (e.g., anemia, pulmonary embolism).

Decreased PO2 in inspired air (choice B) is seen at high altitudes and when the settings are wrong during
artificial ventilation.

Hypoventilation of central origin (choice C) is seen in morphine and barbiturate overdose.

Hypoventilation of peripheral origin (choice D) is seen in poliomyelitis and chest trauma.

28 comments:

Anonymous said...

What's up, constantly i used to check webpage posts here early in the daylight, for the reason that i love to gain knowledge of more and more.

Also visit my website; linked internet site

Anonymous said...

I think that is one of the such a lot important information for me.
And i'm happy studying your article. But wanna statement on few common things, The web site style is ideal, the articles is really great : D. Just right task, cheers

Feel free to visit my blog post; south padre island vacation deals

Anonymous said...

But Thеre are ways tο trickster in the аs the popular
add-ins Internet bгowѕers the heartbeat and JAVA besidеs аllowed the ontogenesis οf moгe сomplex
online games.

Here is my webѕite; game

Anonymous said...

In Train bang uou Ηold to travel to the depthѕ of of сitizеnry
Have gοt sеveral reаsons tо why thеy
ρlay the online games. lastly, Thithеr be seеn from T.
V. aԁs to T. V. shows South Green did а solid eрisode on it Seаrch at the ѵiԁеo department foг a bried good
eхample.

Feel fгee to viѕit my web-site; game

Anonymous said...

This Blog is a blog baseԁ game ωhich in 1950 and гan on CBS until 1967 and and so in first run ѕynԁicаtion frοm 1968 tо 1975.

Anonymous said...

geotorelxzp bill consolidation loans
best debt consolidation

Anonymous said...

Keep on working, great job!

my page learn the facts here now

Anonymous said...

Do you have a spam problem on this site; I also am a blogger,
and I was wanting to know your situation; we have developed some nice practices and we
are looking to exchange methods with other folks, why not
shoot me an e-mail if interested.

Review my website ... http://ddeath.overminddl1.com/

Anonymous said...

You actually make it seem so easy with your presentation however I find
this topic to be really something which I think I would never understand.
It kind of feels too complicated and extremely broad for me.
I am taking a look ahead on your subsequent submit, I'll try to get the grasp of it!

my web site: published here

Anonymous said...

Very soon this web page will be famous amid
all blog visitors, due to it's good content

My weblog http://cashstructuredsettlement.areavoices.com/

Anonymous said...

I was wondering if you ever considered changing the page layout of your site?
Its very well written; I love what youve got to say.
But maybe you could a little more in the way of content so
people could connect with it better. Youve got an awful lot of text for only having 1 or two pictures.
Maybe you could space it out better?

My homepage Continued

Anonymous said...

Hi, I believe your web site might be having internet browser compatibility issues.

Whenever I take a look at your blog in Safari, it
looks fine however when opening in IE, it's got some overlapping issues. I just wanted to give you a quick heads up! Apart from that, great site!

my site - great post to read

Anonymous said...

Υou are ρerpetuаlly acting аgainst а grade,
you сan arcade games,clasѕic Gamеs Colonnade is a 100% Unloose Online Games. Minecraft is a phenomenal secret plan and we are truly these games are played global by multiple players at the same meter. I merely Afford tribal admiration is coined the Victor and the server restarts. Unloose Online Games jocuri starts kind between toys and things to Bear.

Also visit my weblog; http://dibat.ru/

Anonymous said...

It's hard to come by educated people about this subject, however, you sound like you know what you're talking about!
Thanks

My web-site ... vancouver island vacation homes

Anonymous said...

Please let me know if you're looking for a writer for your weblog. You have some really great articles and I think I would be a good asset. If you ever want to take some of the load off, I'd absolutely love to write some articles for your blog in exchange for a link back to mine.
Please send me an email if interested. Thank you!

Feel free to visit my homepage ... healthy diet plans

Anonymous said...

Howdy would you mind sharing which blog platform you're working with? I'm going to start my own blog soon but I'm having a difficult time selecting between BlogEngine/Wordpress/B2evolution and Drupal. The reason I ask is because your design seems different then most blogs and I'm
looking for something completely unique. P.
S Apologies for getting off-topic but I had to ask!

Here is my webpage: make money from home

Anonymous said...

Wonderful post! We are linking to this particularly great content on our site.
Keep up the great writing.

Here is my webpage - Xxx Games

Anonymous said...

My spouse and I stumbled over here from a different website and thought
I might as well check things out. I like what I see so now
i am following you. Look forward to checking out your web
page again.

Also visit my web-site - psn code

Anonymous said...

Excellent post. I was checking continuously this blog and I'm impressed! Extremely helpful information specially the last part :) I care for such info much. I was looking for this certain information for a very long time. Thank you and best of luck.

My homepage ... virtapay to lr

Anonymous said...

Hello would you mind letting me know which web host you're working with? I've loaded your blog in 3 completely different web
browsers and I must say this blog loads a lot faster then
most. Can you suggest a good web hosting provider at a fair price?
Thank you, I appreciate it!

Also visit my web-site - psn code generator

Anonymous said...

Wonderful, what a web site it is! This weblog provides useful data to us, keep it up.


Here is my homepage - Virta pay

Anonymous said...

I know this if off topic but I'm looking into starting my own blog and was wondering what all is needed to get setup? I'm assuming having a blog like yours would cost a pretty
penny? I'm not very web smart so I'm not 100% sure. Any recommendations
or advice would be greatly appreciated. Appreciate it

Here is my site: pdf password remover

Anonymous said...

I like this web site very much, Its a rattling nice
office to read and receive information.
"Reason is not measured by size or height, but by principle."
by Epictetus.

Review my website ... twitter hack

Anonymous said...

Hey very nice blog!! Man .. Excellent .. Wonderful .
. I'll bookmark your blog and take the feeds additionally�I am satisfied to search out so many helpful information right here in the publish, we want develop extra techniques in this regard, thank you for sharing.

My page adf.ly

Anonymous said...

Currently it seems like Expression Engine is the top blogging platform out there right now.

(from what I've read) Is that what you are using on your blog?

Here is my page youtube bot

Anonymous said...

Having read this I believed it was really enlightening.
I appreciate you spending some time and energy to put this informative article together.
I once again find myself spending a significant amount of time both reading and leaving comments.

But so what, it was still worthwhile!

My homepage :: hack Twitter Account

Ahitra Gaethon said...

As a sign of gratitude for how my husband was saved from Lymes, I decided to reach out to those still suffering from this.
My husband suffered from Lymes and it was really tough and heartbreaking for me because he was my all and the symptoms were terrible, we tried various therapies prescribed by our neurologist but none could cure him. I searched for a cure and I saw testimony of so many people who were cured from Lymes , and so many others with similar body problems, and they left the contact of this doctor who had the herbal cure to Lymes. I never imagined Lymes had a cure not until I contacted him and he assured me my husband will be fine. I got the herbal medication he recommended and my husband used it and in one months he was fully okay even up till this moment he is so full of life.Lymes has a cure and it is a herbal cure contact the doctor for more info on drituaherbalcenter@gmail.com Dr Itua can cure Herpes,Hiv,Cancer,ALS,Copd,HPV,Lupus,MS,Diabetes, and other disease talk to Dr Itua on how to get the medication. Thanks for reading my testimony . 

ASpeak said...

I was asked by a medical doctor Nicola Whitehill to purchase Dr Itua herbal medicine for Scleroderma as June is Scleroderma month. It took a lot to go through and remember that I have been through a lot! I have learned a lot about herbal medicines and my fight to have a healthy lifestyle! I have recommended Dr Itua herbal medicines to a lot of people so that they can see that the fight may be hard at times, but it is all worth it! You are worth the fight, so don't ever give up and continue to be your own advocate and you may need to switch doctors to get the best care for your body! Keep your head up and keep moving forward despite the obstacle that you may be on! God only gives you what He can handle, so put your faith, trust, and hope in Him and ask Him to show you the direction in which He wants you to go! By researching and finding what you are putting in or on your body can help tremendously and help you to use Food as Medicine to help heal your gut!
Dr Itua has all types of herbal medicines to cure all kinds of disease such as Herpes,Diabetes,HPV,Copd,Als,Ms,HIV,Cancers,hepatitis,Parkinson,Infertility and other human disease and infections you may have been going through in your life Dr Itua will prepare you a permanent cure.
Dr Itua herbal center email contact: drituaherbalcenter@gmail.com. www.drituaherbalcenter.com